Site Loader

Содержание

Амплитудное значение напряжения в цепи переменного тока

♦Переменный электрический ток в нашей бытовой электросети представляет собой синусоиду, как на рисунке 1.

Напряжение меняет свою величину от 0 до + Umax и от 0 до — Umax . Полный цикл этих изменений называется периодом.
Период измеряется в секундах и обозначается буквой Т .
Количество периодов переменного тока за 1 секунду, есть частота f .
Частота переменного тока f измеряется в герцах .

f = 1 / T.

Например.
Частота в нашей электрической сети 50 Гц . Период этих колебаний будет равен:

T = 1 / f = 1 / 50 = 0,02 сек.

Наибольшее значение изменяющегося переменного напряжения – тока называется амплитудным значением или амплитудой.

Umax = Ua и Imax = Ia

За один период напряжение принимает эти значения два раза: + Ua и — Ua .

♦ Если подключить в цепь переменного напряжения какую-нибудь активную нагрузку, например паяльник, в цепи потечет переменный электрический ток, так же принимающий значения +Ia и — Ia , и повторяющий форму синусоиды.
На нагрузке выделяется электрическая мощность в виде тепла. Неважно какой ток течет в цепи — переменный или постоянный. Выделение тепла не зависит от направления тока в цепи.
Выделенное тепло будет равно той энергии, которую затрачивает электрический ток при прохождении по сопротивлению нагрузки.
Введено понятие действующего значения переменного напряжения Uд и тока Iд.

Действующее значение переменного тока — это такое значение величины постоянного тока, который проходя по сопротивлению нагрузки за тот же промежуток времени, выделит такое же количество тепла, что и переменный ток.

♦ Переменный ток оказывает такое же тепловое действие, как и постоянный ток, если амплитуда синусоидального переменного тока превышает величину постоянного тока в 1,41 раз .
Следовательно действующее (или эффективное) значение переменного тока будет равно:

Iд = Ia / 1,41 = 0,707 Ia. – действующее значение переменного тока

Uд = Ua / 1,41 = 0,707 Ua — действующее значение переменного напряжения

На все эти теоретические размышления можно посмотреть иначе!

♦Имеем синусоиду переменного напряжения длительностью в 1 период как на рисунке 1 .
После выпрямительных диодов оно принимает вид как на рисунке 2.

Нижняя половинка синусоиды перевернута вверх, чтобы удобнее было представить процесс преобразования.

♦На рисунке приняты обозначения:

Um = Ua = 1 — амплитудное значение величины переменного напряжения. Значение Ua примем за единицу.

Из формулы приведенной выше Uд = 1 / 1,41 = 0,707 — действующее напряжение равно 0,707 от амплитудного значения Ua = 1.
Заштрихованная часть синусоиды обозначает затраченную на нагревание паяльника электрическую энергию. В промежутках между половинками синусоид ток по цепи не протекает, а следовательно и не выделяется электрическая мощность.

♦Проведем линию, обозначающую Uд = 0,707.
Она отсекает верхнюю часть половинок синусоид.
Если эти отсеченные вершинки синусоиды уложить в провалы между полупериодами, получится полностью заполненная площадь соответствующая значениям постоянного напряжения U и тока I.
Получается, что мощность синусоидального переменного тока с амплитудными значениями Ua и Ia равна мощности действующего значения Uд и Iд переменного тока и равна мощности постоянного тока со значениями
U и I
.
Одна и та же электрическая мощность, выраженная в трех видах.

P = Ua х Ia = Uд х Iд = U х I

♦ Электрические приборы для измерения переменного напряжения и тока отградуированы на отображение действующих значений Uд и Iд .
В нашей бытовой электросети действующее, эффективное, напряжение переменного тока Uд равно 220 вольт .
Максимальное, амплитудное значение напряжения в сети равно:
Um = Ua = Uд х 1,41 = 220 х 1,41 = 310,2 вольт.

Процесс поэтапного преобразования переменного напряжения в пульсирующее напряжение, а затем в постоянное напряжение, наблюдается в схемах выпрямителей.

♦Переменный электрический ток в нашей бытовой электросети представляет собой синусоиду, как на рисунке 1.

Напряжение меняет свою величину от 0 до + Umax и от 0 до — Umax . Полный цикл этих изменений называется периодом.
Период измеряется в секундах и обозначается буквой Т .
Количество периодов переменного тока за 1 секунду, есть частота f .
Частота переменного тока f измеряется в герцах .

f = 1 / T.

Например.
Частота в нашей электрической сети 50 Гц . Период этих колебаний будет равен:

T = 1 / f = 1 / 50 = 0,02 сек.

Наибольшее значение изменяющегося переменного напряжения – тока называется амплитудным значением или амплитудой.

Umax = Ua и Imax = Ia

За один период напряжение принимает эти значения два раза: + Ua и — Ua .

♦ Если подключить в цепь переменного напряжения какую-нибудь активную нагрузку, например паяльник, в цепи потечет переменный электрический ток, так же принимающий значения +Ia и — Ia , и повторяющий форму синусоиды.
На нагрузке выделяется электрическая мощность в виде тепла. Неважно какой ток течет в цепи — переменный или постоянный. Выделение тепла не зависит от направления тока в цепи.
Выделенное тепло будет равно той энергии, которую затрачивает электрический ток при прохождении по сопротивлению нагрузки.

Введено понятие действующего значения переменного напряжения Uд и тока Iд.

Действующее значение переменного тока — это такое значение величины постоянного тока, который проходя по сопротивлению нагрузки за тот же промежуток времени, выделит такое же количество тепла, что и переменный ток.

♦ Переменный ток оказывает такое же тепловое действие, как и постоянный ток, если амплитуда синусоидального переменного тока превышает величину постоянного тока в 1,41 раз .
Следовательно действующее (или эффективное) значение переменного тока будет равно:

Iд = Ia / 1,41 = 0,707 Ia. – действующее значение переменного тока

Uд = Ua / 1,41 = 0,707 Ua — действующее значение переменного напряжения

На все эти теоретические размышления можно посмотреть иначе!

♦Имеем синусоиду переменного напряжения длительностью в 1 период как на рисунке 1 .
После выпрямительных диодов оно принимает вид как на рисунке 2.

Нижняя половинка синусоиды перевернута вверх, чтобы удобнее было представить процесс преобразования.

♦На рисунке приняты обозначения:

Um = Ua = 1 — амплитудное значение величины переменного напряжения. Значение

Ua примем за единицу.

Из формулы приведенной выше Uд = 1 / 1,41 = 0,707 — действующее напряжение равно 0,707 от амплитудного значения Ua = 1.
Заштрихованная часть синусоиды обозначает затраченную на нагревание паяльника электрическую энергию. В промежутках между половинками синусоид ток по цепи не протекает, а следовательно и не выделяется электрическая мощность.
♦Проведем линию, обозначающую Uд = 0,707.
Она отсекает верхнюю часть половинок синусоид.
Если эти отсеченные вершинки синусоиды уложить в провалы между полупериодами, получится полностью заполненная площадь соответствующая значениям постоянного напряжения U и тока I.


Получается, что мощность синусоидального переменного тока с амплитудными значениями Ua и Ia равна мощности действующего значения Uд и Iд переменного тока и равна мощности постоянного тока со значениями U и I .
Одна и та же электрическая мощность, выраженная в трех видах.

P = Ua х Ia = Uд х Iд = U х I

♦ Электрические приборы для измерения переменного напряжения и тока отградуированы на отображение действующих значений Uд и Iд .
В нашей бытовой электросети действующее, эффективное, напряжение переменного тока Uд равно 220 вольт .
Максимальное, амплитудное значение напряжения в сети равно:

Um = Ua = Uд х 1,41 = 220 х 1,41 = 310,2 вольт.

Процесс поэтапного преобразования переменного напряжения в пульсирующее напряжение, а затем в постоянное напряжение, наблюдается в схемах выпрямителей.

  • 5 — 9 классы
  • Физика
  • 5 баллов

Амплитудное значение напряжения в цепи переменного тока Um=10В.Определите напряжение в момент времени t=T/6 (ответ 5 В)

«>

StudyPort.Ru — Механические и электромагнитные колебания

Страница 1 из 6

4. Колебания и волны

1. Гармонические колебания величины s описываются уравнением s = 0,02 cos (6πt + π/3), м. Определите: 1) амплитуду колебаний; 2) циклическую частоту; 3) частоту колебаний; 4) период колебаний.

2. Запишите уравнение гармонического колебательного движения точки, совершающей колебания с амплитудой A = 8 см, если за t = 1 мин совершается n = 120 колебаний и начальная фаза колебаний равна 45°.

3. Материальная точка совершает гармонические колебания с амплитудой A = 4 см и периодом T = 2 с. Напишите уравнение движения точки, если ее движение начинается из положения x0 = 2 см.

4. Точка совершает гармонические колебания с периодом T = 6 с и начальной фазой, равной нулю. Определите, за какое время, считая от начала движения, точка сместится от положения равновесия на половину амплитуды.

5. Напишите уравнение гармонического колебания точки, если его амплитуда A = 15 см, максимальная скорость колеблющейся точки vmax = 30 см/с, начальная фаза φ = 10°.

6. Точка совершает гармонические колебания по закону x = 3 cos (πt/2 + π/8), м. Определите: 1) период T колебаний: 2) максимальную скорость Vmax точки; 3) максимальное ускорение amax точки.

7. Точка совершает гармонические колебания с амплитудой A = 10 см и периодом T = 5 с. Определите для точки: 1) максимальную скорость; 2) максимальное ускорение.

8. Скорость материальной точки, совершающей гармонические колебания, задается уравнением v(t) = -6 sin 2 πt, м/с. Запишите зависимость смещения этой точки от времени.

9. Материальная точка совершает колебания согласно уравнению x = A sin ωt. В какой-то момент времени смещение точки x1 = 15 см. При возрастании фазы колебания в два раза смещение x2 оказалось равным 24 см. Определите амплитуду A колебания.

10. Материальная точка совершает гармонические колебания согласно уравнению x = 0,02 cos (πt + π/2), м. Определите: 1) амплитуду колебаний; 2) период колебаний; 3) начальную фазу колебаний; 4) максимальную скорость точки; 5) максимальное ускорение точки; 6) через сколько времени после начала отсчета точка будет проходить через положение равновесия.

11. Определите максимальные значения скорости и ускорения точки, совершающей гармонические колебания с амплитудой A = 3 см и периодом T = 4 с.

12. Материальная точка, совершающая гармонические колебания с частотой ν = 1 Гц, в момент времени t = 0 проходит положение, определяемое координатой х0 = 5 см, со скоростью v0 = -15 см/с. Определите амплитуду колебаний.

13. Тело массой m = 10 г совершает гармонические колебания по закону х = 0,1 cos(4πt + π/4), м. Определите максимальные значения: 1) возвращающей силы; 2) кинетической энергии.

14. Материальная точка массой m = 50 г совершает гармонические колебания согласно уравнению x = 0,1 cos 3πt/2, м. Определите: 1) возвращающую силу F для момента времени t = 0,5 с; 2) полную энергию Е точки.

15. Материальная точка массой m = 20 г совершает гармонические колебания по закону x = 0,1 cos(4πt + π/4), м. Определите полную энергию Е этой точки.

16. Полная энергия E гармонически колеблющейся точки равна 10 мкДж, а максимальная сила Fmax, действующая на точку, равна -0,5 мН. Напишите уравнение движения этой точки, если период T колебаний равен 4 с, а начальная фаза φ = π/6.

17. Определите отношение кинетической энергии T точки, совершающей гармонические колебания, к ее потенциальной энергии П, если известна фаза колебания.

18. Определите полную энергию материальной точки массой m, колеблющейся по закону x = A cos(ω0t + φ).

19. Груз, подвешенный к спиральной пружине, колеблется по вертикали с амплитудой A = 8 см. Определите жесткость k пружины, если известно, что максимальная кинетическая энергия Tmax груза составляет 0,8 Дж.

20. Материальная точка колеблется согласно уравнению х = A cos ωt, где A = 5 см и ω = π/12 с-1. Когда возвращающая сила F в первый раз достигает значения -12 мН, потенциальная энергия П точки оказывается равной 0,15 мДж. Определите: 1) этот момент времени t; 2) соответствующую этому моменту фазу ωt.

3. Поведение катушки индуктивности в цепи переменного тока.

Сопротивление катушки постоянному току назовем активным и обозначим через R. Если катушку индуктивности включать в цепь переменного тока, то вследствие периодического изменения силы тока в катушке возникает ЭДС самоиндукции, препятствующая приложенному напряжению. Это приводит к уменьшению силы тока в катушке, иначе говоря, ее сопротивление становится больше, чем активное. То есть, катушка будет обладать не только активным, но и реактивным (индуктивным) сопротивлением RL.

В дальнейшем ограничимся рассмотрением наиболее простого случая, когда R<< RL. При этом индуктивное сопротивление легко рассчитывается.

Пусть через катушку индуктивностью L протекает переменный ток величиной

(4)

При этом на концах катушки возникает напряжение U, что численно равное ЭДС самоиндукции.

(5)

Для переменного тока I, протекающего по постоянному сопроти­влению R, согласно закону Ома можно записать:

(6)

Сравнивая (5) и (6), приходим к выводу, что катушка индуктивности обладает реактивным сопротивлением, равным:

(7)

Таким образом, реактивное сопротивление катушки пропорционально ее индуктивности и частоте переменного тока. Сравнивая (4) и (5), можно прийти к выводу, что колебания напряжения на катушке индуктивности опережает по фазе на π/2 колебания тока в той же катушке. Запишем (5) в виде:

(8)

где U — амплитудное значение напряжения на концах катушки индуктивности. Тогда, сравнивая (5) и (8) и учитывая (7), можно записать, что:

(9)

(10)

Эти формулы позволяют рассчитывать индуктивность катушки по

известному значению U0 и I0, и будут использовать при определении индуктивности методом ампеометра-вольтметра.

Экспериментальная часть

Принципиальная схема для определения индуктивности методом амперметра-вольтметра приведена на рисунке 1.

Установка состоит из источника переменного напряжения частоты (звукового генератора), нагруженного на последовательно соединенные активное сопротивление R и магазин индуктивностей L.

Магазин индуктивностей позволяет получить переменное значение индуктивности в пределах от 0 до 0,1 гн. Величина индуктивности подсчитывается по формуле:

(11)

где n – номер положения переключателя магазина индуктивностей.

Вольтметр VR измеряет амплитудное значение напряжений U0R на активном сопротивлении. Полное отклонение стрелки вольтметра соответствует напряжению в 25 в. Активное сопротивление R равно 10000 Ом. Следовательно, зная U0R, можно рассчитать амплитудное значение тока I0, протекающего по цепи.

(12)

Вольтметр VL. измеряет амплитудное значение напряжения U0R на катушке индуктивности. Полное отклонение стрелки соответствует 10 вольтам. Зная U0R и I0, а также частоту переменного напряжения, можно определить индуктивность катушки по формуле:

(13)

Таким образом, пользуясь источником переменного напряжения с известной частотой , зная амплитудное значение тока I0 , протекающего по катушке и амплитудное значение напряжения на катушке индуктивности U0L по формуле (13) можно рассчитать значение индуктивности этой катушки. Такой метод определения индуктивности носит название метода амперметра-вольтметра.

Переменный ток, генератор переменного тока. Действующее, амплитудное значения напряжения, силы тока в цепи. Тесты онлайн, куосы по физике, подготовка к ЦТ

Всего вопросов: 10

Вопрос 1. Вольтметр переменного тока, включенный в сеть, показывает напряжение 220 В. Найдите максимальное значение напряжения Umax в сети.

Вопрос 2. Напряжение в цепи переменного тока изменяется по закону . Чему равно амплитудное значение напряжения?

Вопрос 3. Зависимость силы тока от времени в цепи имеет вид . Чему равна частота колебаний тока в цепи?

Вопрос 4. Плоский прямоугольный проводящий виток площадью вращается в однородном магнитном поле, совершая 5 об/с вокруг оси, перпендикулярной полю. Как изменяется ЭДС индукции в витке с течением времени, если индукция магнитного поля Тл и в начальный момент времени плоскость витка перпендикулярна магнитному полю? Рассчитайте амплитудное значение ЭДС в витке.

Вопрос 5. Рамка вращается в однородном магнитном поле. ЭДС индукции, возникающая в рамке, изменяется по закону . Определите время одного оборота рамки.

Вопрос 6. Прямоугольная рамка площадью 100 см2 имеет витки в количестве 200 и вращается в однородном магнитном поле, модуль индукции которого 0,02 Тл. Ось вращения рамки перпендикулярна силовым линиям магнитного поля. Амплитудное значение ЭДС индукции, возникающей в рамке, равно 0,5 В. Определите циклическую частоту вращения рамки.

Вопрос 7. Полагая, что напряжение в сети изменяется по закону синуса и начальная фаза , определите мгновенное значение напряжения в момент времени с. Действующее напряжение 220 В, частота 50 Гц.

Вопрос 8. В начальный момент времени напряжение на клеммах генератора переменного тока равно амплитудному, U0=100 В. Частота переменного тока 50 Гц. Определите напряжение на клеммах генератора через с.

Вопрос 9. Определите количество теплоты (кДж), которое выделяется за время 10 мин в кипятильнике сопротивлением 110 Ом, включенном в сеть переменного тока, напряжение в которой изменяется по закону

Вопрос 10. Квадратная рамка со стороной 10 см вращается в однородном магнитном поле с угловой скоростью 300 рад/с. Модуль индукции магнитного поля 20 мТл. Сопротивление рамки 10 Ом, ось вращения рамки перпендикулярна к линиям магнитной индукции. Определите максимальную силу тока (мА) в рамке.

%d0%b0%d0%bc%d0%bf%d0%bb%d0%b8%d1%82%d1%83%d0%b4%d0%bd%d0%be%d0%b5%20%d0%b7%d0%bd%d0%b0%d1%87%d0%b5%d0%bd%d0%b8%d0%b5 — с русского на все языки

Все языкиАбхазскийАдыгейскийАфрикаансАйнский языкАканАлтайскийАрагонскийАрабскийАстурийскийАймараАзербайджанскийБашкирскийБагобоБелорусскийБолгарскийТибетскийБурятскийКаталанскийЧеченскийШорскийЧерокиШайенскогоКриЧешскийКрымскотатарскийЦерковнославянский (Старославянский)ЧувашскийВаллийскийДатскийНемецкийДолганскийГреческийАнглийскийЭсперантоИспанскийЭстонскийБаскскийЭвенкийскийПерсидскийФинскийФарерскийФранцузскийИрландскийГэльскийГуараниКлингонскийЭльзасскийИвритХиндиХорватскийВерхнелужицкийГаитянскийВенгерскийАрмянскийИндонезийскийИнупиакИнгушскийИсландскийИтальянскийЯпонскийГрузинскийКарачаевскийЧеркесскийКазахскийКхмерскийКорейскийКумыкскийКурдскийКомиКиргизскийЛатинскийЛюксембургскийСефардскийЛингалаЛитовскийЛатышскийМаньчжурскийМикенскийМокшанскийМаориМарийскийМакедонскийКомиМонгольскийМалайскийМайяЭрзянскийНидерландскийНорвежскийНауатльОрокскийНогайскийОсетинскийОсманскийПенджабскийПалиПольскийПапьяментоДревнерусский языкПортугальскийКечуаКвеньяРумынский, МолдавскийАрумынскийРусскийСанскритСеверносаамскийЯкутскийСловацкийСловенскийАлбанскийСербскийШведскийСуахилиШумерскийСилезскийТофаларскийТаджикскийТайскийТуркменскийТагальскийТурецкийТатарскийТувинскийТвиУдмурдскийУйгурскийУкраинскийУрдуУрумскийУзбекскийВьетнамскийВепсскийВарайскийЮпийскийИдишЙорубаКитайский

 

Все языкиАнглийскийНемецкийНорвежскийКитайскийИвритФранцузскийУкраинскийИтальянскийПортугальскийВенгерскийТурецкийПольскийДатскийЛатинскийИспанскийСловенскийГреческийЛатышскийФинскийПерсидскийНидерландскийШведскийЯпонскийЭстонскийТаджикскийАрабскийКазахскийТатарскийЧеченскийКарачаевскийСловацкийБелорусскийЧешскийАрмянскийАзербайджанскийУзбекскийШорскийРусскийЭсперантоКрымскотатарскийСуахилиЛитовскийТайскийОсетинскийАдыгейскийЯкутскийАйнский языкЦерковнославянский (Старославянский)ИсландскийИндонезийскийАварскийМонгольскийИдишИнгушскийЭрзянскийКорейскийИжорскийМарийскийМокшанскийУдмурдскийВодскийВепсскийАлтайскийЧувашскийКумыкскийТуркменскийУйгурскийУрумскийЭвенкийскийБашкирскийБаскский

ВЫНУЖДЕННЫЕ ЭЛЕКТРОМАГНИТНЫЕ КОЛЕБАНИЯ. ПЕРЕМЕННЫЙ ТОК. ИНДУКТИВНОСТЬ И ЕМКОСТЬ В ЦЕПИ ПЕРЕМЕННОГО ТОКА. МОЩНОСТЬ ПЕРЕМЕННОГО ТОКА. ТРАНСФОРМАТОР

Вынужденными колебаниями называют такие колебания, которые вызываются действием на систему внешних сил, периодически изменяющихся с течением времени. В случае электромагнитных колебаний такой внешней силой является периодически изменяющаяся э.д.с. источника тока.

Отличительные особенности вынужденных колебаний: вынужденные колебания — незатухающие колебания; частота вынужденных колебаний равна частоте внешнего периодического воздействия на колебательную систему, т.е., в данном случае, равна частоте изменения э.д.с. источника тока.

Амплитуда вынужденных колебаний зависит от частоты изменения э.д.с. источника тока. Для вынужденных колебаний характерно явление электрического резонанса, при котором амплитуда вынужденных колебаний становится максимальной. Это физическое явление наблюдается при совпадении частоты изменения э.д.с. источника тока с собственной частотой колебаний данного контура, т.е.:

, (1)

где: i — мгновенное значение тока, т.е. его значение в момент времени t = 0;

J0 — амплитудное или максимальное значение силы тока;

 — частота изменения тока, численно равная частоте изменения э.д.с. источника тока.

Мгновенным или амплитудным значениями тока и напряжения на практике пользоваться неудобно. Амперметры и вольтметры в цепи переменного тока измеряют так называемые действующие или эффективные значения переменного тока, которые связаны с амплитудными значениями тока по формулам:

, (4)

. (5)

Действующими значениями силы тока и напряжения переменного тока называют значения этих величин для такого постоянного тока, который на том же активном сопротивлении выделяет за время, равное периоду Т переменного тока, такое же количество теплоты, как и данный переменный ток.

Источником переменного тока является генератор переменного тока, физический принцип действия которого основан на равномерном вращении с угловой скоростью  плоской рамки площадью S, состоящей из N витков, в однородном магнитном поле с индукцией В. При этом рамку пронизывает переменный магнитный поток:

, (6)

где: Ф0 — максимальное значение магнитного потока;

 — угол между нормалью к рамке и вектором магнитной индукции В;

- угловая скорость вращения рамки.

Согласно закону электромагнитной индукции, в рамке будет возбуждаться мгновенное значение э.д.с., изменяющееся по закону:

, (7)

где: e — мгновенное значение э.д.с.;

0 — амплитудное значение э.д.с.;

 — угловая скорость вращения рамки.

В общем случае цепь переменного тока представляет собой колебательный контур:

Напряжение на зажимах источника тока U меняется по гармоническому закону с частотой изменения э.д.с. генератора переменного тока.

Существует принципиальное отличие электрического сопротивления цепи переменного тока по сравнению с электрическим сопротивлением цепи постоянного тока, связанное с преобразованиями электрической энергии в другие виды энергии.

Устройства, в которых электрическая энергия полностью и необратимо преобразуется в другие виды энергии, называют активными нагрузками, а электрические сопротивления этих устройств — активными сопротивлениями. В цепи постоянного тока существуют только активные нагрузки.

Устройства, в которых не происходит необратимого превращения электрической энергии в другие виды энергии, называют реактивными нагрузками, а их сопротивления — реактивными сопротивлениями. Реактивные сопротивления в цепи переменного тока имеют конденсатор и катушка индуктивности, которые соответственно называют емкостным xc сопротивлением и индуктивным сопротивлением xL. При этом конденсатор имеет только реактивное сопротивление, а катушка индуктивности, помимо реактивного сопротивления, обладает еще активным сопротивлением. Реактивные сопротивления вычисляются по формулам:

, (8)

, (9)

где: С — емкость конденсатора;

L — индуктивность катушки;

 — частота изменения э.д.с. источника тока.

Если в цепи переменного тока реактивной нагрузки нет или ее сопротивление пренебрежимо мало по сравнению с активным сопротивлением цепи, то колебания силы тока совпадают по фазе с колебаниями напряжения и происходят с частотой и фазой колебаний э.д.с. источника тока:

, (10)

, (11)

. (12)

Цепь переменного тока, которая не содержит конденсатора и активное сопротивление которой ничтожно мало по сравнению с индуктивным сопротивлением, называется цепью переменного тока с индуктивным сопротивлением. В такой цепи колебания напряжения на катушке опережает колебания силы тока на , т.е.:

, (13)

. (14)

Цепь переменного тока, которая не имеет индуктивного сопротивления и активное сопротивление которой пренебрежимо мало по сравнению с емкостным сопротивлением, называется цепью переменного тока с емкостным сопротивлением. В такой цепи колебания силы тока опережают колебания напряжения на :

, (15)

. (16)

Для амплитудного и действующего значений переменного тока справедлив закон Ома:

, (17)

, (18)

, (19)

где величина R называется полным сопротивлением цепи переменного тока.

Количество теплоты Q, выделяющееся на активном сопротивлении, вычисляется по закону Джоуля-Ленца:

. (20)

Величина преобразованной электрической энергии в другие виды энергии определяется мощностью переменного тока. Так как — сила тока и напряжение — переменные величины, то и мощность в цепи переменного тока является переменной величиной. Поэтому имеет смысл говорить только о мгновенном значении мощности , или о среднем значении мощности за период Т изменения переменного тока, вычисляемой по формуле:

. (21)

Мощность называют активной мощностью. Множитель называют коэффициентом мощности, где:  — сдвиг по фазе между колебаниями силы тока и напряжения. Коэффициент мощности вычисляется по формуле:

. (22)

Для преобразования переменного тока одонго напряжения в переменный ток другого напряжения при той же частоте используют устройство, называемое трансформатором. Трансформатор представляет собой систему, состоящую из двух обмоток (катушек), связанных одним сердечником. Если первоначально катушка содержит N1 витков, а вторичная — N2 витков, то коэффициент трансформации k вычисляется по формуле:

, (23)

где 1 и 2 — э.д.с. индукции в первичной и вторичной обмотках.

Если падение напряжения на активном сопротивлении первичной обмотки трансформатора ничтожно мало, то: и . Тогда:

, (24)

где U1 и U2 — напряжение на первичной и вторичной обмотках трансформатора.

К.п.д. трансформатора называют отношение мощности Р2, отдаваемой вторичной обмоткой, к мощности Р1, подводимой к первичной обмотке:

. (25)

К.п.д. современных трансформаторов очень высок — 97-98 %. Поэтому по закону сохранения энергии мощность тока в первичной обмотке практически равна мощности тока во вторичной обмотке: Р1 Р2. Отсюда следует, что: J1U1 J2U2.

Тогда формулу (24) можно записать в виде:

, (26)

где: J1, J01 — действующее и амплитудное значения тока в первичной обмотке;

J2, J02 -действующее и амплитудное значения тока во вторичной обмотке.

Методические указания к решению задач

Среди задач на вынужденные электромагнитные колебания можно выделить следующие пять групп задач: задачи на вращение плоской рамки в однородном магнитном поле и получение переменной э.д.с.; задачи, требующие знание общих уравнений гармонических колебаний, э.д.с., силы тока и напряжения в цепи; задачи на расчет элементов цепи переменного тока с использованием закона Ома; задачи на тепловое действие переменного тока и расчет мощности тока; задачи о трансформаторах.

Для решения первого из перечисленных типов задач используются формулы (6), (7). При затруднении применения этих формул надо повторить закон электромагнитной индукции, а также основные характеристики вращательного и колебательного движений, связь между ними.

Задачи на составление общих уравнений гармонических колебаний э.д.с., силы тока или напряжения в цепи переменного тока требует не только хорошего знания уравнений (2), (3), (10) — (16), но и четкого понимания того, что такое мгновенное, амплитудное и действующее (эффективное) значения э.д.с., силы тока и напряжения и какая существует связь между амплитудными и действующими значениями этих физических величин (4), (5).

Часто встречается и обратная задача — по заданным уравнениям путем сравнительного анализа с вышеназванными уравнениями определить амплитудное значение колеблющейся величины, фазу между колебаниями силы тока и напряжения, который можно вычислить, через тригонометрические функции, например, по формуле (22).

Большое количество задач связано с расчетом цепи переменного тока с помощью закона Ома. Они похожи на расчет цепи постоянного тока. Трудность лишь в том, что надо внимательно оценивать нагрузку цепи переменного тока в отношении ее сопротивления. В частных случаях, когда в цепи надо учитывать только активное, емкостное или индуктивное сопротивления, задачи решаются довольно просто с учетом формул (8), (9). В общем случае надо использовать закон Ома в виде (17), (18) с учетом полного сопротивления цепи (19).

Задачи на расчет цепи переменного тока предполагают также знание явления электрического резонанса и умения рассчитывать резонансную частоту (1).

Количество теплоты, выделяемое в цепи переменного тока, рассчитывается по закону Джоуля-Ленца (20). Надо быть особенно внимательным при расчете мощности, выделяемой на активной нагрузке цепи переменного тока, которая в общем случае вычисляется с учетом сдвига по фазе между колебаниями силы тока и напряжения (21).

Распространены также задачи технического содержания, в частности, связанные с трансформаторами. Для успешного решения подобных задач надо повторить устройство и принцип действия трансформатора. Расчеты в таких задачах, в основном, связаны со знанием коэффициента трансформации (24), (26) и умения вычислять к.п.д. трансформатора (25).

Примеры решения задач

Задача 1. Определить амплитудное и действующее значения переменной э.д.с., возникающей в рамке при ее вращении с постоянной скоростью в однородном магнитном поле, если при угле поворота рамки на 45 град. мгновенное значение э.д.с. равно 156 В.

Дано:

 = 45 град;

e = 156 В

Найти:

0 — ? д — ?

Мгновенное значение э.д.с., возникающей в рамке при равномерном вращении в однородном магнитном поле, прямо пропорционально минусу угла поворота плоскости рамки относительно направления линий индукции магнитного поля, т.е., согласно формуле (7) имеем:

.

Отсюда: ; .

Учитывая связь между амплитудными и действующими значениями физических величин (4), (5), получим:

.

Задача 2. Магнитный поток в рамке, состоящей из 1000 витков и равномерно вращающейся в однородном магнитном поле, изменяется по закону . Найти зависимость мгновенной э.д.с. индукции, возникающей в рамке, от времени. Определить амплитудное и действующее значения э.д.с., период и частоту тока.

N = 1000

Найти:

e(t),0,д,T,- ?

Мгновенное значение э.д.с., возникающей в каждом витке e1 равно первой производной от магнитного потока по времени, взятой со знаком минус. Следовательно: ; .

Учитывая тот факт, что имеется не один виток, а N витком получим:

; .

Анализируя полученное выражение и сравнивая его с формулой (7), получим: .

Соответственно, действующее значение э.д.с.:

.

Для нахождения периода Т и частоты  надо учесть связь между круговой частотой  и периодом Т: и связь круговой частоты  с линейной частотой : .

Окончательно получим:

; ; ; .

Задача 3. В сеть переменного тока с действующим напряжением 220 В и частотой 50 Гц включен контур, состоящий из резистора сопротивлением 100 Ом, конденсатора емкостью 35,4 мкФ и катушки с индуктивностью 0,7 Гн. Написать уравнения U(t) и i(t) зависимости напряжения и силы тока от времени.

Дано:

Uд = 220 В

 = 50 Гц

Ra = 100 Ом

С = 35,4 мкФ

L = 0,7 Гн

Найти:

i(t) — u(t) —

Чтобы успешно справиться с этой задачей, надо помнить, что в цепи переменного тока колебания силы тока и напряжения происходят по синусоидальному закону с одинаковой круговой частотой , но со сдвигом по фазе  (2), (3):

; .

Амплитудное значение напряжения U0 и круговая частота  легко находятся из данных условия задачи. Действительно: ; ; ; .

Для определения амплитудного значения силы тока J0 надо воспользоваться законом Ома с учетом формул (19, 4):

; .

Сдвиг фаз между колебаниями силы тока и внешнего напряжения определяется соотношением (22):

; ; .

С учетом полученных числовых значений J0, U0, ,  уравнения зависимости силы тока i и напряжения U от времени t получим:

; .

Следовательно, в данной цепи напряжение опережает по фазе силу тока на 0,3  рад, или на 54 град.

Задача 4. Найти падение напряжения на резисторе, конденсаторе и катушке, а также определить частоту переменного тока, при которой в цепи наступит резонанс, воспользовавшись данными предыдущей задачи.

Дано:

Uд = 220 В

Ra = 100 Ом

С = 35,4 мкФ

L = 0,7 Гн

Найти:

UR,UC,UL,0

Расчет падения напряжения на различных элементах цепи переменного тока принципиально ничем не отличается от подобного расчета в цепи постоянного тока. Так как элементы цепи соединены последовательно, то:

; ; ; ,

где: .

Окончательно получим:

; ; ; ; ; .

Для нахождения резонансной частоты воспользуемся формулой (1) и формулой :

; .

Откуда: ; .

Задача 5. Конденсатор емкостью 0,5 мкФ включен в сеть переменного тока. Определить период колебаний переменного тока, если емкостное сопротивление конденсатора равно 20 Ом.

Дано:

С = 510-5 Ф

ХС = 20 Ом

Найти:

Т —

Из формулы емкостного сопротивления (8) с учетом выразим Т: .

Откуда: ; .

 

Задача 6. Катушка индуктивностью 15 мГн включена в сеть промышленного переменного тока. Определить ее индуктивное сопротивление.

Дано:

L=1,510-2Гн

 = 50 Гц

Найти:

ХL — ?

В данной задаче надо помнить о том, что частота промышленного тока 50 Гц.

Далее по формуле (9) с учетом — находим:

; ;

Задача 7. При включении конденсатора сила тока в цепи переменного тока с частотой 50 Гц и напряжением 220 В равна 0,14 А. Определите емкость конденсатора. Активным сопротивлением цепи можно пренебречь.

Дано:

 = 50 Гц

Uд = 220 В

Jд = 0,14 А

Найти:

С —

Воспользуемся законом Ома для цепи переменного тока (18): .

Так как: и индуктивное сопротивление , то закон Ома для цепи с емкостной нагрузкой будет в виде:

.

Отсюда: ; . С учетом соотношения — перепишем емкостное сопротивление ХС в виде:

. Откуда: ; .

Задача 8. В сеть переменного тока с частотой 50 Гц и напряжением 220 В включена катушка с индуктивностью 60 мГн и ничтожно малым активным сопротивлением. Найдите действующее и амплитудное значение силы тока в цепи катушки.

Дано:

 = 50 Гц

Uд = 220 В

L = 6010-3 = = 0,06 Гн

Найти:

Jд, J0 — ?

Индуктивное сопротивление катушки найдем по формуле (9):

; .

Закон Ома для цепи с индуктивной нагрузкой при и имеет вид: .

Отсюда: .

Амплитудное значение силы тока вычислим по формуле (4):

; .

Задача 9. Конденсатор емкостью 24 мкФ и катушка индуктивностью 0,05 Гн с активным сопротивлением 100 Ом подключены последовательно к источнику переменного тока, в котором напряжение на зажимах равно 120 В при частоте 50 Гц. Определить действующие значения силы тока в цепи напряжения на конденсаторе и катушке.

Дано:

С = 210-5 Ф

R = 100 Ом

L = 0,05 Гн

 = 50 Гц

Uд = 120 В

Найти:

Jд, UC, UL — ?

В данной задаче имеются все элементы цепи переменного тока, поэтому для нахождения силы тока надо воспользоваться законом Ома в общем виде (18) с учетом формулы (19):

.

Учитывая связь — , окончательно получаем:

.

Для вычисления действующего напряжения на конденсаторе необходимо силу тока в цепи умножить на емкостное сопротивление конденсатора:

; .

Действующее значение напряжения на катушке равно силе тока, умноженной на полное сопротивление катушки с учетом того, что катушка индуктивности имеет как индуктивное сопротивление, так и активное сопротивление:

; .

Задача № 10. В сеть переменного тока с действующим напряжением 110 В включены последовательно конденсатор с емкостью 510-5 Ф, катушки индуктивностью 200 мГн и активным сопротивлением 4 Ом. Определить амплитудное значение силы тока в цепи, если частота переменного тока 100 Гц, а также частоту переменного тока, при которой в данном контуре наступит резонанс напряжений.

Дано:

Uд = 110 В

С = 510-5 Ф

L = 0,2 Гн

R = 4 Ом

 = 100 Гц

Найти:

J0, рез — ?

По закону Ома (17) — (19) амплитудное значение силы тока: .

С учетом: и окончательно получаем:

; .

При резонансе частота вынужденных колебаний переменного тока совпадает с частотой собственных колебаний контура. Отсюда:

; .

Задача № 11. Электропечь, сопротивление которой 22 Ом, питается от генератора переменного тока. Определить количество теплоты, выделяемое печью за 1 час, если амплитудное значение силы тока 10 А.

Дано:

R = 22 Ом

t = 3,6104 с

J0 = 10 А

Найти:

Q — ?

Количество теплоты, выделяемое в цепи переменного тока на активном сопротивлении, можно вычислять по закону Джоуля-Ленца, используя действующие значения силы тока или напряжения. В данном случае: .

Учитывая связь — , окончательно получаем: ; ; .

Задача 12. На входе линии электропередачи, потребляющей мощности 100 кВт, подается напряжение 220 В. Определить потери мощности в проводах, сопротивление которых 0,01 Ом при сдвиге фазы 37 град. между колебаниями силы тока и напряжения.

Дано:

P = 105 B

Uд = 220 В

R = 0,01 Ом

 = 37 град

Найти:

Р1 — ?

Мощность, потребляемая активным сопротивлением, как и в случае постоянного тока, вычисляется по формуле:

.

Действующее значение силы тока может быть вычислено по формуле (2): , откуда: .

Окончательно получаем: ; .

Задача 13. Трансформатор имеет коэффициент трансформации 18. Напряжение на первичной обмотке, содержащей 450 витков, равно 220 В. Определите напряжение на вторичной обмотке и число витков в ней.

Дано:

k = 18

N1 = 450

U1 = 220 B

Найти:

U2, N2 — ?

Решение:

Воспользуемся формулой для коэффициента трансформации (24):

.

Откуда:

; ; ; .

Задача 14. Сила тока в первичной обмотке трансформатора J1 = 15000 А и напряжение на ее зажимах U1 = 11000 В. Сила тока во вторичной обмотке J2 = 1500 А. Определить напряжение на зажимах вторичной обмотки трансформатора, если к.п.д. равен 96 %.

Дано:

J1 = 15000 A

U1 = 11000 B

J2 = 1500 A

к.п.д. = 0,96

Найти:

U2 — ?

Коэффициент полезного действия трансформатора по определению вычисляется по формуле (25):

К.п.д. = ,

где: ; .

Отсюда к.п.д. = .

Окончательно получаем:

; .

Задача 15. Первичная обмотка трансформатора имеет 500 витков, вторичная — 50 витков. В первичной обмотке сила тока изменяется по закону: . По какому закону изменяется сила тока во вторичной обмотке в рабочем режиме трансформатора Считать, что токи в первичной и вторичной обмотках совершают колебания в одинаковых фазах.

Дано:

N1 = 500

N2 = 50

Найти:

i2(t) — ?

Так как колебания тока в обмотках совершаются в одинаковых фазах, то законы изменения силы тока будут отличаться только амплитудными значениями силы тока.

Для того, чтобы найти амплитудное значение тока во вторичной обмотке трансформатора, надо воспользоваться выражениями для коэффициента трансформации (24), (26):

.

Отсюда получим: ; .

Теперь можно записать закон изменения силы тока во вторичной обмотке: .

ЭЛЕКТРОМАГНИТНЫЕ ВОЛНЫ

Краткая теория

Электромагнитные волны — электромагнитные колебания, распространяющиеся в пространстве с течением времени. Источником электромагнитных волн является открытый колебательный контур.

В электромагнитной волне происходят колебания напряженности электрического поля и индукции магнитного поля. Вектора и перпендикулярны друг другу и направлению распространения волны, т.е. электромагнитные волны — поперечные волны.

Основной характеристикой электромагнитной волны является ее скорость, величина которой зависит от свойств среды, в которой волна распространяется. В вакууме скорость электромагнитных волн равна скорости света — 3108 м/с. Следующей важной характеристикой волны является длина волны — . Длина волны — это расстояние между ближайшими друг к другу точками, колеблющимися в одинаковых фазах, с другой стороны, — это расстояние, на которое перемещается волна с заданной скоростью v за время t, равное периоду колебаний T:

. (1)

Учитывая связь между периодом Т и частотой колебаний — , формулу (1) можно переписать в виде:

. (2)

В вакууме: . (3)

С помощью электромагнитных волн осуществляется радиосвязь, радиовещание, радиолокация, космическая связь и т.д.

Методические указания к решению задач

Решение задач в школьном курсе физики сводится, в основном, к двум типам. В первую очередь, это расчет длины волны, излучаемой колебательным контуром, через частоту собственных колебаний в данном контуре.

Для вычисления собственной частоты колебаний применяется формула:

. (4)

Для вычисления периода собственных колебаний применяют формулу .Томсона: . (5)

Если излучение происходит в воздухе, то скорость электромагнитных волн можно считать равной скорости эл. м. волн в вакууме — с. Если о среде нет никаких дополнительных сведений, то в большинстве случаев подразумевается вакуум и в качестве скорости берется та же скорость — с.

Возможна и обратная задача — по известной излучаемой длине волны требуется вычислить какой-нибудь из параметров колебательного контура.

Приемником электромагнитных волн также является колебательный контур, резонирующий на определенную длину волны. Поэтому при решении задач на прием электромагнитных волн необходимо помнить о явлении электрического резонанса и условии, при котором это явление наблюдается:

. (6)

Второй тип задач связан, в основном, с применением электромагнитных волн, например, в радиолокации. Надо помнить, что в этом случае пользуются одним из свойств электромагнитных волн — отражением электромагнитных волн от границы раздела двух различных сред.

Примеры решения задач

Задача 1.Определить длину электромагнитных волн в воздухе, излучаемых колебательным контуром с емкостью 3 нФ и индуктивностью 0,012 Гн. Активное сопротивление контура принять равным нулю.

Дано:

С = 310-9 Ф

L = 0,012 Гн

Ra = 0

v=c= 3108м/с

Найти:

 — ?

Решение:

Согласно определению длина волны определяется по формуле (1): .

Подставляя значение для периода собственных колебаний (5) в формулу (1), получим:

;

.

Задача 2. Колебательный контур излучает в воздухе электромагнитные волны длиной 300 м. Определить индуктивность колебательного контура, если его емкость равна 5 мкФ. Активное сопротивление контура не учитывать.

Дано:

l = 300 м

С = 510-6 Ф

Ra = 0

С = 3108 м/с

Найти:

L — ?

Решение:

Решая совместно два уравнения (1) и (5), получаем:

и .

Отсюда получим:

; ; ;

.

Задача 3. На какую длину волны будет резонировать колебательный контур, в котором индуктивность катушки 8 мкГн, а емкость конденсатора 20 нФ

Дано:

L = 810-6 Гн

С = 210-8 Ф

с = 3108 м/с

Найти:

рез — ?

Решение:

Электрический резонанс наступает в контуре при условии, если частота колебаний электрического поля и магнитного поля в электромагнитной волне совпадает с частотой электромагнитных колебаний в контуре, т.е. по формуле (6) имеем:

.

Тогда длина волны легко рассчитывается по формуле (3):

,

.

Задача 4. Радиопередатчик работает на частоте 6 МГц. Сколько волн укладывается на расстоянии 100 км по направлению распространения радиосигнала

Дано:

 = 6106 Гц

S = 105 м

c = 3108 м/с

Найти:

n — ?

Решение:

На данном расстоянии очевидно должно укладываться целое число длин волн, т.е.: .

Так как длина волны вычисляется по формуле (3), то окончательно получим: ; .

Задача 5. Радиолокатор работает в импульсном режиме. Частота повторения импульсов 1700 Гц, длительность импульса 0,8 мкс. Найти максимальную и минимальную дальность обнаружения цели данным локатором.

Дано:

f = 1700 Гц

 = 0,8 мкс =

= 810-7 с

с = 3108 м/с

Найти:

l, L — ?

Решение:

Для того, чтобы по положению отраженного импульса на экране радиолокатора можно было судить о расстоянии до цели, необходимо, чтобы отраженный импульс пришел не ранее, чем через время , и не позднее, чем через время , после начала посылки прямого импульса.

Следовательно, минимальное расстояние до цели:

; ,

где:  — время возвращения сигнала после отражения от цели, учитывая, что сигнал проходит расстояние l дважды.

Максимальное расстояние до цели:

; .

 

 

Действующее значение тока формула. Большая энциклопедия нефти и газа

Как известно, переменная э.д.с. индукции вызывает в цепи переменный ток. При наибольшем значении э.д.с. сила тока будет иметь максимальное значение и наоборот. Это явление называется совпадением по фазе. Несмотря на то что значения силы тока могут колебаться от нуля и до определенного максимального значения, имеются приборы, с помощью которых можно замерить силу переменного тока.

Характеристикой переменного тока могут быть действия, которые не зависят от направления тока и могут быть такими же, как и при постоянном токе. К таким действиям можно отнести тепловое. К примеру, переменный ток протекает через проводник с заданным сопротивлением. Через определенный промежуток времени в этом проводнике выделится какое-то количество тепла. Можно подобрать такое значение силы постоянного тока, чтобы на этом же проводнике за то же время выделялось этим током такое же количество тепла, что и при переменном токе. Такое значение постоянного тока называется действующим значением силы переменного тока.

В данное время в мировой промышленной практике широко распространен трехфазный переменный ток , который имеет множество преимуществ перед однофазным током. Трехфазной называют такую систему, которая имеет три электрические цепи со своими переменными э.д.с. с одинаковыми амплитудами и частотой, но сдвинутые по фазе относительно друг друга на 120° или на 1/3 периода. Каждая такая цепь называется фазой.

Для получения трехфазной системы нужно взять три одинаковых генератора переменного однофазного тока, соединить их роторы между собой, чтобы они не меняли свое положение при вращении. Статорные обмотки этих генераторов должны быть повернуты относительно друг друга на 120° в сторону вращения ротора. Пример такой системы показан на рис. 3.4.б.

Согласно вышеперечисленным условиям, выясняется, что э.д.с., возникающая во втором генераторе, не будет успевать измениться, по сравнению с э.д.с. первого генератора, т. е. она будет опаздывать на 120°. Э.д.с. третьего генератора также будет опаздывать по отношению ко второму на 120°.

Однако такой способ получения переменного трехфазного тока весьма громоздкий и экономически невыгодный. Чтобы упростить задачу, нужно все статорные обмотки генераторов совместить в одном корпусе. Такой генератор получил название генератор трехфазного тока (рис. 3.4.а). Когда ротор начинает вращаться, в каждой обмотке возникает


а) б)

Рис. 3.4. Пример трехфазной системы переменного тока

а) генератор трёхфазного тока; б) с тремя генераторами;

изменяющаяся э.д.с. индукции. Из-за того что происходит сдвиг обмоток в пространстве, фазы колебаний в них также сдвигаются относительно друг друга на 120°.

Для того чтобы подсоединить трехфазный генератор переменного тока к цепи, нужно иметь 6 проводов. Для уменьшения количества проводов обмотки генератора и приемников нужно соединить между собой, образовав трехфазную систему. Данных соединений два: звезда и треугольник. При использовании и того и другого способа можно сэкономить электропроводку.

Соединение звездой

Обычно генератор трехфазного тока изображают в виде 3 статорных обмоток, которые располагаются друг к другу под углом 120°. Начала обмоток принято обозначать буквами А, В, С , а концы — X, Y, Z . В случае, когда концы статорных обмоток соединены в одну общую точку (нулевая точка генератора), способ соединения называется «звезда». В этом случае к началам обмоток присоединяются провода, называемые линейными (рис. 3.5 слева).

Точно так же можно соединять и приемники (рис. 3.5., справа). В этом случае провод, который соединяет нулевую точку генератора и приемников, называется нулевой. Данная система трехфазного тока имеет два разных напряжения: между линейным и нулевым проводами или, что то же самое, между началом и концом любой обмотки статора. Такая величина называется фазным напряжением (). Поскольку цепь трехфазная, то линейное напряжение будет в v3 раз больше фазного, т. е.: Uл = v3Uф.

Соединение треугольником.

Рисунок 3.6. Пример соединения треугольником

При использовании данного способа соединения конец X первой обмотки генератора подключают к началу В второй его обмотки, конец Y второй обмотки — к началу С третьей обмотки, конец Z третьей обмотки — к началу А первой обмотки. Пример соединения показан на рис. 3.6. При данном способе соединения фазных обмоток и подключении трехфазного генератора к трехпроводной линии линейное напряжение по своему значению сравнивается с фазным: Uф = Uл

Контрольные вопросы

1. Перечислите основные параметры, характеризующие переменный ток.

2. Дайте определение частоты и единицы её измерения.

3. Дайте определение амплитуды и единицы её измерения.

4. Дайте определение периода и единицы его измерения.

5. Отличие простейшего генератора трёхфазного тока от генератора однофазного тока.

6. Что такое фаза?

7. Что представляет собой ротор генератора трёхфазного тока?

8. Почему сдвинуты по фазе обмотки статора генератора трёхфазного тока?

9. Особенность симметричной системы трёх фаз.

10. Принцип соединения фазных обмоток трёхфазных генераторов и трансформаторов по схеме «звезда».

11. Принцип соединения фазных обмоток трёхфазных генераторов и трансформаторов по схеме «треугольник».

3.2. Виды сопротивлений в цепях переменного тока

В цепях переменного тока сопротивления разделяют на активные и реактивные.

В активных сопротивлениях , включенных в цепь переменного тока, электрическая энергия преобразуется в тепловую. Активным сопротивлением R обладают, например, провода электрических линий, обмотки электрических машин и т.д.

В реактивных сопротивлениях электрическая энергия, вырабатываемая источником, не расходуется. При включении реактивного сопротивления в цепь переменного тока возникает лишь обмен энергией между ним и источником электрической энергии. Реактивное сопротивление создают индуктивности и ёмкости.

Если не учитывать взаимное влияние отдельных элементов электрической цепи, то в общем случае электрическая цепь синусоидального тока может быть представлена тремя пассивными элементами: активным сопротивлением R, индуктивностью L и емкостью C.

Активное сопротивление в цепи переменного тока .

При включении в цепь переменного тока активного сопротивления, ток и напряжение совпадают по фазе (рис. 3.7) и изменяются по одному и тому же cинусоидальному закону: u=U m sinωt . Они одновременно достигают своих максимальных значений и одновременно проходят через нуль (рис. 3.7.б).

Для цепи переменного тока, содержащей только активное сопротивление, закон Ома имеет такую же форму, как и для цепи постоянного тока: I=U/R.

Электрическая мощность р в цепи с активным сопротивлением в любой момент времени равна произведению мгновенных значений силы тока i и напряжения u : p=ui .

Рисунок 3.7. Схема включения в цепь переменного тока активного сопротивления R (a), кривые тока i , напряжения u и мощности p (б) и векторная диаграмма.

Из графика видно, что изменение мощности происходит с двойной частотой по отношению к изменению тока и напряжения, т.е. один период изменения мощности соответствует половине периода изменения тока и напряжения. Все значения мощности положительные, это означает, что энергия передается от источника к потребителю.

Средняя мощность Рcp , потребляемая активным сопротивлением, P=UI=I 2 R – это и есть активная мощность.

Под индуктивностью L будем понимать элемент электрической цепи (катушку индуктивности, потерями которой можно пренебречь), способный запасать энергию в своём магнитном поле, который не имеет активного сопротивления и ёмкостиС (рис.3.8).

При включении в цепь переменного тока индуктивности, изменяющийся ток непрерывно индуцирует в ней э.д.с. самоиндукции e L = LΔi/Δt, где Δi/Δt – скорость изменения тока.

Когда угол ωt равен 90° и 270° скорость изменения тока Δi/Δt =0, поэтому э.д.с. e L =0.

Скорость изменения тока будет наибольшей, когда угол ωt равен 0°, 180° и 360°. В эти минуты времени э.д.с. имеет наибольшее значение.

Кривая мощности представляет собой синусоиду, которая изменяется с двойной частотой по сравнению с частотой изменения тока и напряжения. Мощность имеет положительные и отрицательные значения, т.е. возникает непрерывный колебательный процесс обмена энергией между источником и индуктивностью.

Рисунок 3.8. Схема включения в цепь переменного тока индуктивности (а), кривые тока i , напряжения u , э.д.с. e L (б) и векторная диаграмма (в)

Э.д.с. самоиндукции согласно правилу Ленца направлена так, чтобы препятствовать изменению тока. В первую четверть периода, когда ток увеличивается, э.д.с. имеет отрицательное значение (направлена против тока).

Во вторую четверть периода, когда ток уменьшается, э.д.с. имеет положительное значение (совпадает по направлению с током).

В третью четверть периода ток меняет своё направление и увеличивается, поэтому э.д.с. направлена против тока и имеет положительное значение.

В четвёртую четверть периода ток уменьшается и э.д.с. самоиндукции стремится поддержать прежнее положение тока и имеет отрицательное значение. В результате ток отстает от напряжения по фазе на угол 90 О.

Сопротивление катушки или проводника переменному току, вызванное действием э.д.с. самоиндукции, называется индуктивным сопротивлением Х L [Ом]. Индуктивное сопротивление не зависит от материала катушки и от площади поперечного сечения проводника.

В цепях переменного тока катушки индуктивности соединяют последовательно и параллельно.

При последовательном соединении катушек эквивалентная индуктивность и эквивалентное индуктивное сопротивление X L э будут равны:

Lэ=L 1 +L 2 +… X L э=X L 1 +X L 2 +…

При параллельном соединении катушек:

1/Lэ=1/L 1 +1/L 2 +… 1/X L э=1/X L 1 +1/X L 2 +…

Контрольные вопросы

1. Какие виды сопротивления в цепях переменного тока Вы знаете?

2. Что значит активное сопротивление?

3. Что такое реактивное сопротивление?

4. Какие элементы цепи создают реактивное сопротивление?

5. Что такое активная мощность?

1. Дайте определение индуктивности.

2. Что происходит в первую четверть периода колебательного процесса обмена энергией между источником и индуктивностью?

3. Что происходит во вторую четверть периода колебательного процесса обмена энергией между источником и индуктивностью?

4. Дайте определение индуктивного сопротивления.

3.3. Конденсаторы. Ёмкость в цепи переменного тока

Конденсатор – устройство, способное накапливать электрические заряды.

Простейший конденсатор представляет собой две металлические пластины (электроды), разделенные диэлектриком.

Каждый конденсатор характеризуется номинальной емкостью и допустимым напряжением. Напряжение конденсатора указывают на корпусе, и превышать его нельзя. Конденсаторы различаются формой электродов (плоский), типом диэлектрика и ёмкостью (постоянной и переменной).

Силу переменного тока (напряжения) можно охарактеризовать при помощи амплитуды. Однако амплитудное значение тока непросто измерить экспериментально. Силу переменного тока удобно связать с каким-либо действием, производимым током, не зависящим от его направления. Таковым является, например, тепловое действие тока. Поворот стрелки амперметра, измеряющего переменный ток, вызывается удлинением нити, которая нагревается при прохождении по ней тока.

Действующим илиэффективным значением переменного тока (напряжения) называется такое значение постоянного тока, при котором на активном сопротивлении выделяется за период такое же количество теплоты, как и при переменном токе.

Свяжем эффективное значение тока с его амплитудным значением. Для этого рассчитаем количество теплоты, выделяемое на активном сопротивлении переменным током за время, равное периоду колебаний. Напомним, что по закону Джоуля-Ленца количество теплоты, выделяющееся на участке цепи cсопротивлениемприпостоянном токеза время, определяется по формуле

. Переменный ток можно считать постоянным только в течение очень малых промежутков времени

. Поделим период колебанийна очень большое число малых промежутков времени

. Количество теплоты

, выделяемое на сопротивленииза время

:

. Общее количество теплоты, выделяемое за период, найдется суммированием теплот, выделяемых за отдельные малые промежутки времени, или, другими словами, интегрированием:


.

Сила тока в цепи изменяется по синусоидальному закону


,


.

Опуская вычисления, связанные с интегрированием, запишем окончательный результат


.

Если бы по цепи шёл некоторый постоянный ток , то за время, равное, выделилось бы тепло

. По определению постоянный ток, оказывающий такое же тепловое действие, что и переменный, будет равен эффективному значению переменного тока

. Находим эффективное значение силы тока, приравнивая теплоты, выделяемые за период, в случаях постоянного и переменного токов


(4.28)

Очевидно, точно такое же соотношение связывает эффективное и амплитудное значения напряжения в цепи с синусоидальным переменным током:


(4.29)

Например, стандартное напряжение в сети 220 В – это эффективное напряжение. По формуле (4.29) легко посчитать, что амплитудное значение напряжения в этом случае будет равно 311 В.

4.4.5. Мощность в цепи переменного тока

Пусть на некотором участке цепи с переменным током сдвиг фаз между током и напряжением равен , т.е. сила тока и напряжение изменяются по законам:


,

.

Тогда мгновенное значение мощности, выделяемой на участке цепи,

Мощность изменяется со временем. Поэтому можно говорить лишь о ее среднем значении. Определим среднюю мощность, выделяемую в течение достаточно длительного промежутка времени (во много раз превосходящего период колебаний):

С использованием известной тригонометрической формулы


.

Величину

усреднять не нужно, так как она не зависит от времени, следовательно:


.

За длительное время значение косинуса много раз успевает измениться, принимая как отрицательные, так и положительные значения в пределах от (1) до 1. Понятно, что среднее во времени значение косинуса равно нулю


, поэтому

(4.30)

Выражая амплитуды тока и напряжения через их эффективные значения по формулам (4.28) и (4.29), получим


. (4.31)

Мощность, выделяемая на участке цепи с переменным током, зависит от эффективных значений тока и напряжения и сдвига фаз между током и напряжением . Например, если участок цепи состоит из одного только активного сопротивления, то

и

. Если участок цепи содержит только индуктивность или только ёмкость, то

и

.

Объяснить среднее нулевое значение мощности, выделяемой на индуктивности и ёмкости можно следующим образом. Индуктивность и ёмкость лишь заимствуют энергию у генератора, а затем возвращают её обратно. Конденсатор заряжается, а затем разряжается. Сила тока в катушке увеличивается, затем снова спадает до нуля и т. д. Именно по той причине, что на индуктивном и ёмкостном сопротивлениях средняя расходуемая генератором энергия равна нулю, их назвали реактивными. На активном же сопротивлении средняя мощность отлична от нуля. Другими словами провод с сопротивлением при протекании по нему тока нагревается. И энергия, выделяемая в виде тепла, назад в генератор уже не возвращается.

Если участок цепи содержит несколько элементов, то сдвига фаз может быть иным. Например, в случае участка цепи, изображенного на рис. 4.5, сдвиг фаз между током и напряжением определяется по формуле (4.27).

Пример 4.7. К генератору переменного синусоидального тока подключён резистор с сопротивлением. Во сколько раз изменится средняя мощность, расходуемая генератором, если к резистору подключить катушку с индуктивным сопротивлением

а) последовательно, б) параллельно (рис. 4.10)? Активным сопротивлением катушки пренебречь.

Решение. Когда к генератору подключено одно только активное сопротивление, расходуемая мощность


(см. формулу (4.30)).

Рассмотрим цепь на рис. 4.10, а. В примере 4.6 было определено амплитудное значение силы тока генератора:

. Из векторной диаграммы на рис. 4.11,а определяем сдвиг фаз между током и напряжением генератора


.

В результате средняя расходуемая генератором мощность


.

Ответ: при последовательном включении в цепь индуктивности средняя мощность, расходуемая генератором, уменьшится в 2 раза.

Рассмотрим цепь на рис. 4.10,б. В примере 4.6 было определено амплитудное значение силы тока генератора

. Из векторной диаграммы на рис. 4.11,б определяем сдвиг фаз между током и напряжением генератора


.

Тогда средняя мощность, расходуемая генератором

Ответ: при параллельном включении индуктивности средняя мощность, расходуемая генератором, не изменяется.

Cтраница 2

Действующим значением силы тока I называется сила постоянного тока, выделяющего в проводнике за то же время такое же количество теплоты, что и переменный ток.  

Как видно из рисунка, в каждый момент времени величины напряжения и силы тока принимают различные значения. Поэтому, чтобы судить о величине силы тока и напряжения переменного тока, пользуются действующим значением силы тока и напряжения. Чтобы определить действующее значение силы переменного тока, его приравнивают к силе постоянного тока, которое выделило бы в проводнике такое же количество тепла, как и переменный ток.  

Трансформатор, содержащий в первичной обмотке 300 витков, включен в сеть переменного тока с действующим напряжением 220 В. Вторичная цепь трансформатора питает нагрузку с активным сопротивлением 50 Ом. Найти действующее значение силы тока во вторичной цепи, если падение напряжения во вторичной обмотке трансформатора, содержащей 165 витков, равно 50 В.  

Таким образом, при замене операции извлечения корня сравнением время, за которое интегрируемый сигнал с ГЛИН станет равен интегралу от квадрата измеренной силы тока, пропорционально действующему значению силы тока. До этого К2 был открыт в течение времени т и пропускал на счетчик СИ импульсы с генератора тактовых импульсов ГТИ. Число импульсов TV / гтит записанное в СЧ, пропорционально действующему значению силы тока. Это число хранится в / 77, а по окончании цикла измерения отображается на ЦИ.  

Как и при механических колебаниях, в случае электрических колебаний обычно нас не интересуют значения силы тока, напряжения и других величин в каждый момент времени. Важны общие характеристики колебаний, такие, как амплитуда, период, частота, действующие значения силы тока и напряжения и средняя мощность. Именно действующие значения силы тока и напряжения регистрируют амперметры и вольтметры переменного тока.  

Рх o jjFr В слУчае т — н — СУХОЙ лампы применяют способ термометра, подвешиваемого вблизи генераторной лампы, и отмечают его показание. Затем, разрывая цепь колебательного контура генератора, дают на сетку генераторной лампы положительный потенциал до тех пор, пока термометр не по. Беря в последнем случае величины 1а и Еа как исходные, определяем Рх из соотношения Рх1а Еа. Мощность в антенне определяется по ф-ле Рх — / /, где РЯ — мощность eW, ra — активное сопротивление антенны в Q и 1а — действующее значение силы тока в антенне в А. По скольку по современным международным нормам под мощностью передатчика принято понимать мощность в антенне, то упомянутая выше ф-ла определяет одновременно и мощность передатчика.  

Тепловые измерители имеют наиболее широкое практич. Действие тепловых измерителей состоит в удлинении тонкой проволоки при нагревании ее проходящим по ней переменным током высокой частоты. Сам по себе эффект ограничивает пределы применимости таких измерителей токами от нескольких тА до 1 — 3 А в зависимости от материала тонкой проволоки, примененной в измерителе. Применяются сплавы серебра с платиной, платины с иридием и др. Если сплав применяется в виде проволоки, то она имеет диаметр сотых долей мм. При ленте толщина составляет 0 01 мм, ширина 3 мм и длина 25 — 30 мм. Удлинение нити нагреваемым током пропорционально квадрату действующего значения силы тока. Перемещение по шкале измерителя стрелки, связанной с той же проволокой с помощью особой подвижной системы, обычно пропорционально квадратному корню из действующей силы тока. Из-за этого шкалы тепловых измерителей имеют неравномерные интервалы между делениями.  

В данном случае колебания тока являются гармоническими (график колебаний — синусоида) и вынужденными, поскольку параметры колебаний (частота, амплитуда) определяются внешним источником — генератором. Некоторые электротехнические устройства (например, колебательный контур) способны генерировать свободные гармонические колебания электрического тока. По левой ветви рамки — от нас и, поскольку в этом случае через клемму а течет ток в направлении, обратном показанному на рис. 12.1, ее полярность — минус. Поскольку при данном положении рамки сила тока имеет наибольшее значение, фаза колебаний может быть г / 2 или 3 / 2ir, в зависимости от того, какое направление тока в рамке мы принимаем за положительное. Сравнивая формулу (12.1) и заданную зависимость, нетрудно заметить, что 1т 10 А и ш 4тград / с. Далее, используя формулу (12.2), определяем частоту колебаний (отв. Используя закон Джоуля — Ленца (Q I2Rt), определяем действующее значение силы тока (отв.  

Синусоидальное напряжение

— обзор

3.3 Амплитуда и фазовые отношения для элементов схемы

Начнем с резистора . Используя выражения для переменного напряжения и переменного тока и подставляя их в закон Ома, мы можем вывести отношения между пиковыми значениями и начальными фазами для резистора:

(3.8) v (t) = Vmcos (ωt + ϕV),

(3.9) i (t) = Imcos (ωt + ϕI),

(3.10) Vmcos (ωt + ϕV) = Ri (t) = RImcos (ωt + ϕI).

Из последнего уравнения следует, что пиковые и начальные значения фаз тока и напряжения должны быть связаны следующим образом:

Следовательно, очевидно, что напряжение и ток через резистор имеют одинаковый начальный фазовый угол; другими словами, они совпадают по фазе . График переменного напряжения и тока на Рисунке 3.2 иллюстрирует этот факт.

Рисунок 3.2. Напряжение и ток переменного тока в случае резистора.

Теперь рассмотрим случай индуктора . Напомним, что соотношение между напряжением и током на клеммах в случае катушки индуктивности:

Используя это уравнение, мы найдем желаемые соотношения. Сначала возьмем производную тока по времени:

(3.14) ddt (Imcos (ωt + ϕI)) = — ωImsin (ωt + ϕI).

Подставляя (3.1) и (3.14) в (3.13), находим:

(3.15) Vmcos (ωt + ϕV) = — LωImsin (ωt + ϕI).

Используя тригонометрическое тождество

(3.16) cos (α + π2) = — sinα,

, последнее уравнение можно переписать следующим образом:

(3.17) Vmcos (ωt + ϕV) = LωImcos (ωt + ϕI + π2).

Из этого уравнения мы находим, что:

Следовательно, переменное напряжение и ток не совпадают по фазе, и переменное напряжение на катушке индуктивности опережает переменный ток через катушку индуктивности на π / 2 Другой способ выразить пиковое значение и фазу отношения в катушке индуктивности

Последнее выражение предполагает, что ток отстает от напряжения на π / 2.Рисунок 3.3 дает графическое представление переменного напряжения и тока в случае катушки индуктивности.

Рисунок 3.3. Напряжение и ток переменного тока в случае индуктора.

Наконец, рассмотрим случай конденсатора . Напомним, что соотношение между клеммами между током и напряжением в случае конденсатора:

Сначала возьмем производную напряжения по времени:

(3.23) ddt (Vmcos (ωt + ϕV)) = — ωVmsin (ωt + ϕV).

Подставляя (3.23) и (3.5) в (3.22) находим:

(3.24) Imcos (ωt + ϕI) = — CωVmsin (ωt + ϕV).

Используя тригонометрическое тождество (3.16), преобразуем (3.24) следующим образом:

(3.25) Imcos (ωt + ϕI) = CωVmcos (ωt + ϕV + π2).

Из этого выражения мы находим, что пиковые значения и фазы связаны соотношением:

, которое также можно записать следующим образом:

Таким образом, в случае конденсаторов, мы видим, что переменное напряжение отстает от тока на π / 2 или, другими словами, переменный ток опережает переменное напряжение на π / 2.Рисунок 3.4 дает графическое представление переменного напряжения и тока в случае конденсатора.

Рисунок 3.4. Напряжение и ток переменного тока в случае конденсатора.

ПРИМЕР 3.1

Предположим, что у нас есть источник синусоидального напряжения с фиксированной амплитудой В м = 5 В и фиксированной частотой f = 1 МГц. Какой максимальный ток будет через (а) резистор 5 Ом, (б) индуктор 5 мкГн и (в) конденсатор 5 мкФ?

Решение: (a) Из (3.11) Im = Vm / R = 5V / 5Ω = 1 A, (b) из (3.20) Im = Vm / ωL = 5V / (2π × 106 Гц × 5 × 10−6H) = 1 / (2π) A = 159 мА, (c) из (3.26) Im = ωCVm = 2π × 106 Гц × 5 × 10−6F × 5V = 50π A = 157 A.

На основании предыдущего обсуждения мы можем сделать вывод, что напряжения переменного тока на резисторах, индукторах, а конденсаторы приводят к возникновению переменного тока, и наоборот. Этот факт следует из линейности соотношений выводов резисторов, конденсаторов и катушек индуктивности. Следовательно, если все источники напряжения и тока в цепи являются источниками переменного тока с одинаковой частотой, то напряжения и токи повсюду в цепи являются синусоидальными функциями с этой частотой.В следующих нескольких разделах мы представим метод анализа цепей, основанный на этом факте.

[PDF] 45. Пиковое значение переменного тока в устройстве мощностью 1500 Вт составляет 5,4 А. Какое действующее значение напряжения на нем?

1 PHYS Практические проблемы hapters 8- hapter Пиковое значение переменного тока в устройстве мощностью 5 Вт составляет 5,4 А. Что такое …

PHYS 102

Практические проблемы

Главы 18–22

Глава 18. 45.

Пиковое значение переменного тока в устройстве мощностью 1500 Вт равно 5.4 А. Какое среднеквадратичное значение напряжения? Мощность и ток можно использовать для определения пикового напряжения, а затем по пиковому напряжению можно определить среднеквадратичное значение напряжения. I пик 2 (1500 Вт) 2P 2 P = I rmsVrms = Vrms → Vrms = = = 3,9 × 10 VI пик 5,4 A 2

Глава 19. 15.

Восемь огней для рождественской елки мощностью 7,0 Вт подключены последовательно к каждому прочее и к источнику 110 В. Какое сопротивление у каждой лампочки? Каждая лампочка получит одну восьмую от общего напряжения, поэтому Vbulb =

Vtot 8

.Используйте это напряжение и мощность

, рассеиваемую каждой лампочкой, для расчета сопротивления лампы. 2

Pbulb =

20.

Vbulb R

2

→ R =

Vbulb P

2

=

Vtot 64 P

=

(110 В) W)

27 Ом

Каково полное сопротивление цепи, подключенной к батарее на рис. 19-41? Каждое сопротивление имеет R = 2,8 кОм. Резисторы пронумерованы на прилагаемой диаграмме, чтобы облегчить анализ.R1 и R2 включены последовательно с эквивалентным сопротивлением R12 = R + R = 2 R. Эта комбинация работает параллельно с R3, с

R2

R1

R3

R4

−1

1 1  эквивалентное сопротивление R123 =  +  = 23 R. Эта комбинация входит в серию  R 2R  с R4, с эквивалентным сопротивлением R1234 = 23 R + R = 35 R. Эта комбинация работает параллельно с R5 с эквивалентным сопротивлением -1

R5

1 3  5 +  = 8 R.Наконец, эта комбинация включена последовательно с R6, и мы вычисляем конечное эквивалентное сопротивление  R 5R. R12345 = 

Req = 58 R + R = 138 R = 138 (2,8 кОм) = 4,55 кОм

R6

27.

Определите величину и направление токов через R1 и R2 на рис. 19-47 . Поскольку в нижней ветви нет резисторов, можно написать уравнения петли Кирхгофа, которые имеют только один текущий член, что упрощает их решение. Чтобы найти ток через R1, обойдите внешний контур против часовой стрелки, начиная с нижнего левого угла.V3 — I1 R1 + V1 = 0 → I1 =

V3 + V1 R1

=

6,0 В + 9,0 В 22 Ом

= 0,68 А, слева

Чтобы найти ток через R2, обойдите нижнюю петлю против часовой стрелки, начиная с левого нижнего угла. V3 — I 2 R2 = 0 → I 2 =

V3 R2

=

6,0 В 15 Ом

= 0,40 A, слева

50. На рис. 19–56 (как на рис. 19–20а) полное сопротивление составляет 15,0 кОм, а ЭДС батареи — 24,0 В. Если измеренная постоянная времени равна 35.0 мкс, рассчитайте (а) общую емкость цепи и (б) время, необходимое для того, чтобы напряжение на резисторе достигло 16,0 В после замыкания переключателя. (а) Из уравнения. 19-7, произведение RC равно постоянной времени. τ 35,0 × 10–6 с τ = RC → C = = = 2,33 × 10–9 F 3 R

15,0 × 10 Ом

(б) Так как батарея имеет ЭДС 24,0 В, если напряжение на резисторе равно 16,0 В, напряжение на конденсаторе будет 8,0 В. Используйте выражение для напряжения на зарядном конденсаторе.t  V   V  — t / τ −t / τ VC = E (1 — e) → e =  1 — C  → — = ln  1 — C  → τ  E   E   8.0 V   V  −6 −5 t = −τ ln  1 — C  = — (35,0 × 10 с) ln  1 — = 1,42 × 10 с   E   24,0 В  Глава 20. 5.

Сила, действующая на провод с током 8,75 А, составляет максимум 1,28 Н при размещении между полюсными поверхностями магнита. Если полюсные грани имеют диаметр 55,5 см, какова приблизительная напряженность магнитного поля? Используйте уравнение. 20-2. Длина провода в B-поле такая же, как диаметр полюсов.Fmax = IlB → B =

Fmax Il

=

1,28 N

(8,75A) (0,555 м)

= 0,264 T

12.

rr Определите направление B для каждого случая на рис. 20–52, где F представляет собой максимальную магнитную силу r, действующую на положительно заряженную частицу, движущуюся со скоростью v. Правило правой руки, примененное к скорости и магнитному полю, даст направление силы. Используйте это, чтобы определить направление магнитного поля с учетом скорости и силы.(а) вниз (б) внутрь в бумагу (в) вправо

15.

Электрон испытывает наибольшую силу при движении 2,9 x 106 м / с в магнитном поле, когда он движется на север. Сила направлена ​​вверх и имеет величину 7,2 x 10-13 Н. Каковы величина и направление магнитного поля? Магнитное поле можно найти из уравнения. 20-4, а направление определяется правилом правой руки. Помните, что заряд отрицательный. Fmax = qvB → B =

Fmax qv

=

7.2 × 10−13 N

(1,60 × 10

−19

) (

C 2,9 × 106 мс

)

= 1,55 T

Для правила правой руки направление должно быть на восток, приложенный к скорости и магнитному полю, чтобы дать правильное направление силы. 30. Определите величину и направление силы между двумя параллельными проводами длиной 35 м и 6,0 см друг от друга, каждый из которых имеет ток 25 А в одном направлении. Поскольку токи параллельны, сила, действующая на каждый провод, будет притягивающей к другому проводу.Используйте уравнение. 20-7, чтобы рассчитать величину силы. F2 =

36.

(4π × 10−7 Тл ⋅ м A) (25 A) 2 (35 м) = 0,0729 Н, привлекательный µ0 I1 I 2 l2 = 2π d 2π (6,0 × 10−2 м)

Прямой поток протонов проходит через заданную точку пространства со скоростью 1,5 x 109 протонов / с. Какое магнитное поле они создают на расстоянии 2,0 м от луча? Поток протонов представляет собой ток, величина которого определяется умножением скорости протона на заряд протона. Затем используйте уравнение. 20-6 для расчета магнитного поля.Bstream

−7 9 −19 µ0 I (4π × 10 Тл ⋅ м A) (1,5 × 10 протонов с) (1,60 × 10 протонов C) −17 = = = 2,40 × 10 Тл 2π r 2π (2,0 м)

Глава 21. 5.

Петля из проволоки диаметром 12,0 см изначально ориентирована перпендикулярно магнитному полю 1,5 Тл. Петля поворачивается так, чтобы ее плоскость была параллельна направлению поля за 0,20 с. Какова средняя наведенная ЭДС в контуре? Поток меняется, потому что петля вращается. Угол меняется от 0o до 90o. Величина средней наведенной ЭДС определяется формулой.21-2а. E = —

∆Φ B ∆t

= —

AB∆ cos θ ∆t

π (0,060 м) 1,5 T (cos 90o — cos 0o) 2

= —

0,20 с

2

= —

17.

π (0,060 м) 1,5 T (0 — 1) 0,20 с

= 84,8 мВ

На рис. 21-12 штанга движется со скоростью 1,6 м / с, имеет длину 30,0 см и сопротивление 2,5 Ом. Магнитное поле составляет 0,35 Тл, а сопротивление U-образного проводника в данный момент составляет 25,0 Ом.Рассчитайте (а) наведенную ЭДС, (б) ток в U-образном проводнике и (в) внешнюю силу, необходимую для поддержания постоянной скорости стержня в этот момент. (a)

Поскольку скорость перпендикулярна магнитному полю и стержню, мы находим наведенную ЭДС из уравнения. 21-3. E = Blv = (0,35 Тл) (0,300 м) (1,6 мс) = 0,168 В

(b)

Найдите наведенный ток по закону Ома. E 0,168 В −3 I =

(в)

R

=

27,5 Ом

= 6.11 × 10 A

Наведенный ток в стержне будет меньше. Поскольку этот ток находится в восходящем магнитном поле, слева будет магнитная сила. Чтобы стержень двигался, справа должна быть равная внешняя сила, заданная формулой. 20-2.

(

−3

)

−4

F = IlB = 6,11 × 10 A (0,300 м) (0,35 Тл) = 6,41 × 10 Н

20.

Простой генератор используется для генерации пиковое выходное напряжение 24,0 В. Квадратный якорь состоит из 6 обмоток.0 см по бокам и вращается в поле 0,420 Тл со скоростью 60,0 об / с. Сколько витков проволоки нужно намотать на квадратную арматуру? Мы находим количество витков из уравнения. 21-5. Коэффициент, умножающий синусоидальный член, — это пиковое выходное напряжение. Epeak = NBω A → N =

Epeak Bω A

=

24,0 В

(0,420 T) 2π рад

об. (60 об. С) (0,060 м)

2

= 42,1 ≈ 42 петли

32.

Повышающий трансформатор увеличивает 25 В до 120 В.Какой ток во вторичной обмотке по сравнению с первичной обмоткой? Используйте уравнения. 21-6 и 21-7, чтобы связать отношения напряжения и тока. VS

=

VP

42.

NS

;

NP

IS IP

=

NP

VS

NS

=

VP

IP

IS

VP4

VS

25 В 120 В

= 0.208

Сколько витков провода потребуется для создания индуктивности 130 мГн из заполненной воздухом катушки длиной 30,0 см и диаметром 5,2 см? Используйте соотношение для индуктивности соленоида, как показано в Примере 21-14. L =

55.

µ0 N 2 A

→ N =

l

Ll

=

µ0 A

(0,130 H) (0,300 м)

(4π × 10

) (

T ⋅ m A π 2,6 × 10−2 м

)

2

= 3823 витка

На какой частоте будет 2.Конденсатор 40 мкФ имеет реактивное сопротивление 6,70 кОм? Мы находим частоту из уравнения. 21-12b для реактивного сопротивления конденсатора. XC =

66.

1

→ f =

2π fC

1 2π XCC

=

1

(

3

) (100007

−6

Ω 2,40 × 10 F

)

= 9,90 Гц

Резистор 2,5 кОм, соединенный последовательно с катушкой индуктивности 420 мГн, приводится в действие источником переменного тока. На какой частоте импеданс вдвое больше, чем импеданс 60 Гц?

Импеданс определяется формулой.21-15 без емкостного реактивного сопротивления. 2 2 2 2 Z = R + X L = R + (2π fL). 2

2

R + 4π f L = 4  R + 4π 2

2

2

2

2

f =

2

3RHz + 16π

9000

2

2

2

R + 4π f L = 2 R + 4π

Z f = 2 Z 60 →

2

(60 Гц) 2

2

4π L

2

2

(60 Гц) 2

L

=

2

2

(60 Гц)

2

2

L

2 2 2 2 L 2 2 2 L 4 R + 16π (60 Гц) L →

3R 2 2

2

2

4π L

+ 4 (60 Гц) =

3 (2500 Ом) 4π

2

2

(0.42 H)

2

+ 4 (60 Гц)

2

Глава 22. 4.

В электромагнитной волне, бегущей на запад, поле B колеблется вертикально, имеет частоту 80,0 кГц и среднеквадратичную напряженность 6,75 x 10-9 Тл. Каковы частота и среднеквадратичная напряженность электрического поля и каково его направление? (См. Рис. 22-7). Частота двух полей должна быть одинаковой: 80,0 кГц. Среднеквадратичная напряженность электрического поля составляет Erms = cBrms = (3,00 × 108 мс) (6,75 × 10–9 Тл) = 2.03В м.

Электрическое поле перпендикулярно как направлению движения, так и магнитному полю, поэтому электрическое поле колеблется вдоль горизонтальной линии север-юг. 18.

r Поле E в электромагнитной волне в свободном пространстве имеет пик 21,8 мВ / м. Какова средняя скорость, с которой эта волна переносит энергию через единицу площади в единицу времени? Энергия на единицу площади в единицу времени S = ​​12 cε 0 E0 2

=

32.

1 2

(3,00 × 10

8

2

м с) (8.85 × 10−12 C 2 Н м 2) (21,8 × 10−3 В · м) = 6,31 · 10−7 Вт · м2.

Генератор FM-станции с частотой 96,1 МГц имеет индуктивность 1,8 мкГн. Какое значение должно быть у емкости? Находим емкость по резонансной частоте: 1

 1  1  2 f0 =   ;  2π  LC  1

2 1  1   , что дает C = 1,53 × 10−12 F = 1,53 пФ. 96,1 × 106 Гц =  −6 2 π 1,8 × 10 H C    () 

ВОПРОС 1 Пиковое значение переменного напряжения составляет 2 В. Что такое среднеквадратичное значение? (3 знака после запятой) QUEST…

  • Генератор переменного тока выдает среднеквадратичное (не пиковое) напряжение 180 В при 60,0 Гц. …

    Генератор переменного тока выдает среднеквадратичное (не пиковое) напряжение 180 В при 60,0 Гц. Генератор соединен последовательно с катушкой индуктивности 0,50 Гн, конденсатором 6,0-1Ф и резистором на 300 Ом. Каков пиковый (не действующий) ток через резистор? Vrms = 2 Ime (R? + (X2 — Xc) ”), Xı = 26 fL, X.: 9 2fC

  • 2 Для синусоидальной волны, представленной ниже на рисунке 1, уменьшите амплитуду от пика до пика, среднеквадратичное значение…

    2 Для синусоидальной волны, представленной ниже на Рисунке 1, уменьшите размах от пика до пика, среднеквадратичное значение и Рисунок I. 3. Симусоидальное напряжение приложено к резистивному кольцу на Рисунке 2, как показано ниже. Определите () i (d) Ipo (e) на положительном пике () b) 1 Рисунок 2 Определите частоту и период времени для угловой скорости 1256 рад / с? Синусоидальная волна A имеет положительный переход через нуль ส เ 0 °. Синусоидальная волна B имеет положительное пересечение нуля на 20P ….

  • Вопрос 21 10 баллов Генератор переменного тока выдает среднеквадратичное (не пиковое) напряжение 180 В при ат…

    Вопрос 21 10 баллов Генератор переменного тока выдает среднеквадратичное (не пиковое) напряжение 180 В при 60,0 Гц. Генератор соединен последовательно с катушкой индуктивности 0,50 Гн, конденсатором 6,0-4Ф и резистором на 300 Ом. Каков пиковый (не действующий) ток через резистор? (R2 + (x1 — Xo) «), X = 2nfL, X 2016 Vrms = ZIms z = 9 1,30 A 0,325 A 2,60 A 0,65 A

  • 1. Каково значение напряжения VacRMS для сигнала, показанного ниже? 10 В 2 В 2 ….

    1. Каково значение напряжения VacRMS для сигнала, показанного ниже? 10 В 2 В 2.Какому размаху напряжения соответствует переменное напряжение 12 ВАКАМ? 3. В чисто резистивной цепи переменного тока, где 10vacpK-PK подается на резистор 8 Ом, ток равен 4. Полная емкость конденсатора 330HF, подключенного параллельно конденсатору 470 мкФ, равна. Если сигнал переменного тока имеет частоту 125кГц, сколько времени нужно на …

  • Вопрос 5 1 балл Генератор переменного тока выдает пиковое (не среднеквадратичное) напряжение 150 В …

    Вопрос 5 1 балл Генератор переменного тока выдает пиковое (не действующее значение) напряжение 150 В при 60.0 Гц. Генератор соединен последовательно с катушкой индуктивности 35 мГн, конденсатором 45 мкФ и резистором 85-0. Ссылка: Ссылка 23-3 Какое из следующих утверждений относительно этой схемы верно? Фазовый угол для этой схемы положительный. Схема более емкостная, чем индуктивная. Напряжение и ток точно не совпадают по фазе. Напряжение и ток в …

  • Напряжение переменного тока, пиковое значение которого составляет 17 В, составляет 312 Ом. резистор. Какие…

    Напряжение переменного тока, пиковое значение которого составляет 17 В, составляет 312 Ом. резистор.Какое значение среднеквадратичного значения тока в резисторе? Дайте свой ответ 3 сф.

  • 1. Напишите уравнение, описывающее напряжение, подаваемое настенная розетка. (f = 60 Гц, …

    1. Напишите уравнение, описывающее напряжение, подаваемое настенная розетка. (f = 60 Гц, Vrms = 120 В) 2. Каково мгновенное напряжение синусоидальной волны при половина периода? Опишите переменный ток (AC). Объясните, как резисторы, конденсаторы и катушки индуктивности ведут себя в цепи переменного тока. ТРЕБОВАНИЯ К ЗАПИСИ Неофициальные: вопросы — 6 баллов, таблицы данных — 3 балла, вопросы опроса из учебной программы — 1 балл ВВЕДЕНИЕ V До этого момента мы использовали запасы…

  • Вопрос 8 Среднеквадратичное значение напряжения на резисторе 172 составляет 11,8 В. Каково пиковое значение …

    Вопрос 8 Среднеквадратичное значение напряжения на резисторе 172 составляет 11,8 В. Каков пиковый ток, протекающий через резистор? (Ответ дайте в амперах, но без единиц измерения. Допуск на ответ составляет 3%.)

  • D Вопрос 2 10 баллов Трехфазный мостовой выпрямитель потребляет 12 кВт от источника питания с межфазным напряжением 150 В. Коэффициент мощности равен 0.77. Какое максимальное (пиковое) значение тока …

    D Вопрос 2 10 баллов Трехфазный мостовой выпрямитель потребляет 12 кВт от источника питания с межфазным напряжением 150 В. Коэффициент мощности 0,77. Какое максимальное (пиковое) значение тока, подаваемого источником питания? 10 баллов DI Вопрос 3 Полномостовой инвертор прямоугольной формы питает нагрузку RL с сопротивлением R-7 Ом и L-58-мГн. Напряжение постоянного тока составляет 90 В, а частота выходного сигнала — 60 Гц. Какова величина третьей гармоники…

  • Вопрос 11 1 балл Пиковое напряжение источника переменного тока составляет 200 В. Что такое …

    Вопрос 11 1 балл Пиковое напряжение источника переменного тока составляет 200 В. Какое среднеквадратичное напряжение? O 200 VO 282 VO 141 VO 100V 1 балл Вопрос 12 В типичном домашнем переменном напряжении 120 В рассчитайте пиковое напряжение: O 240 VO 120 VO 170 VO 85 В 1 балл Вопрос 13 В последовательной цепи переменного тока индуктивное сопротивление составляет 50 а частота 100 Гц. Какие…

  • 15.2 Простые схемы переменного тока — Университетская физика, том 2

    Задачи обучения

    К концу этого раздела вы сможете:

    • Расшифровка векторных диаграмм и их применение к цепям переменного тока с резисторами, конденсаторами и катушками индуктивности
    • Определите реактивное сопротивление резистора, конденсатора и катушки индуктивности, чтобы понять, как ток в цепи ведет себя по сравнению с каждым из этих устройств

    В этом разделе мы изучаем простые модели источников переменного напряжения, подключенных к трем компонентам схемы: (1) резистору, (2) конденсатору и (3) катушке индуктивности.Мощность, обеспечиваемая источником переменного напряжения, имеет ЭДС, равную

    . v (t) = V0sinωt, v (t) = V0sinωt,

    , как показано на рисунке 15.4. Эта синусоидальная функция предполагает, что мы начинаем регистрировать напряжение, когда оно равно v = 0Vv = 0V в момент времени t = 0s.t = 0s. Может быть задействована фазовая постоянная, которая сдвигает функцию, когда мы начинаем измерять напряжения, подобно фазовой постоянной в волнах, которые мы изучали в Waves. Однако, поскольку мы свободны выбирать, когда мы начинаем исследовать напряжение, мы можем пока игнорировать эту фазовую постоянную.Мы можем измерить это напряжение на компонентах схемы, используя один из двух методов: (1) количественный подход, основанный на наших знаниях схем, или (2) графический подход, который объясняется в следующих разделах.

    Фигура 15.4 (a) Выход v (t) = V0sinωtv (t) = V0sinωt генератора переменного тока. (b) Символ, используемый для обозначения источника переменного напряжения на принципиальной схеме.

    Резистор

    Сначала рассмотрим резистор, подключенный к источнику переменного напряжения. Согласно правилу петли Кирхгофа, мгновенное напряжение на резисторе, показанном на рисунке 15.5 (а) —

    vR (t) = V0sinωtvR (t) = V0sinωt

    , а мгновенный ток через резистор

    . iR (t) = vR (t) R = V0Rsinωt = I0sinωt.iR (t) = vR (t) R = V0Rsinωt = I0sinωt. Фигура 15.5 (а) Резистор, подключенный к источнику переменного напряжения. (b) Ток iR (t) iR (t) через резистор и напряжение vR (t) vR (t) на резисторе. Эти две величины находятся в фазе.

    Здесь I0 = V0 / RI0 = V0 / R — амплитуда изменяющегося во времени тока. Графики iR (t) iR (t) и vR (t) vR (t) показаны на рисунке 15.5 (б). Обе кривые достигают своих максимумов и минимумов в одно и то же время, то есть ток через резистор и напряжение на резисторе синфазны.

    Графические представления фазовых соотношений между током и напряжением часто используются при анализе цепей переменного тока. Такие изображения называются векторными диаграммами . Диаграмма вектора для iR (t) iR (t) показана на рисунке 15.6 (a) с током на вертикальной оси. Стрелка (или вектор) вращается против часовой стрелки с постоянной угловой частотой ω, ω, поэтому мы видим ее в один момент времени.Если длина стрелки соответствует амплитуде тока I0, I0, проекция вращающейся стрелки на вертикальную ось равна iR (t) = I0sinωt, iR (t) = I0sinωt, то есть мгновенному току.

    Фигура 15,6 (a) Векторная диаграмма, представляющая ток через резистор на рисунке 15.5. (b) Векторная диаграмма, представляющая как iR (t) iR (t), так и vR (t) vR (t).

    Вертикальная ось на векторной диаграмме может быть либо напряжением, либо током, в зависимости от исследуемого вектора.Кроме того, на одной векторной диаграмме могут быть изображены несколько величин. Например, как ток iR (t) iR (t), так и напряжение vR (t) vR (t) показаны на диаграмме рисунка 15.6 (b). Поскольку они имеют одинаковую частоту и находятся в фазе, их векторы указывают в одном направлении и вращаются вместе. Относительные длины двух векторов произвольны, потому что они представляют разные величины; однако отношение длин двух векторов может быть представлено сопротивлением, поскольку один является вектором напряжения, а другой — вектором тока.

    Конденсатор

    Теперь рассмотрим конденсатор, подключенный к источнику переменного напряжения. Согласно правилу петли Кирхгофа мгновенное напряжение на конденсаторе на рисунке 15.7 (а) равно

    . vC (t) = V0sinωt. vC (t) = V0sinωt.

    Напомним, что заряд конденсатора определяется выражением Q = CV.Q = CV. Это верно в любое время, измеренное в цикле переменного напряжения. Следовательно, мгновенный заряд конденсатора составляет

    q (t) = CvC (t) = CV0sinωt.q (t) = CvC (t) = CV0sinωt.

    Поскольку ток в цепи — это скорость, с которой заряд входит (или покидает) конденсатор,

    iC (t) = dq (t) dt = ωCV0cosωt = I0cosωt, iC (t) = dq (t) dt = ωCV0cosωt = I0cosωt,

    где I0 = ωCV0I0 = ωCV0 — амплитуда тока.Используя тригонометрическое соотношение cosωt = sin (ωt + π / 2), cosωt = sin (ωt + π / 2), мы можем выразить мгновенный ток как

    iC (t) = I0sin (ωt + π2). iC (t) = I0sin (ωt + π2).

    Разделив V0V0 на I0I0, мы получим уравнение, похожее на закон Ома:

    V0I0 = 1ωC = XC. V0I0 = 1ωC = XC.

    15.3

    Величина XCXC аналогична сопротивлению в цепи постоянного тока в том смысле, что обе величины являются отношением напряжения к току. В итоге у них одна и та же единица — ом. Однако имейте в виду, что конденсатор накапливает и разряжает электрическую энергию, а резистор ее рассеивает.Величина XCXC известна как емкостное реактивное сопротивление конденсатора или сопротивление конденсатора изменению тока. Это обратно пропорционально зависит от частоты источника переменного тока — высокая частота приводит к низкому емкостному реактивному сопротивлению.

    Фигура 15,7 (а) Конденсатор, подключенный к генератору переменного тока. (b) Ток iC (t) iC (t) через конденсатор и напряжение vC (t) vC (t) на конденсаторе. Обратите внимание, что iC (t) iC (t) опережает vC (t) vC (t) на π / 2π / 2 рад.

    Сравнение выражений для vC (t) vC (t) и iC (t) iC (t) показывает, что между ними существует разность фаз π / 2radπ / 2rad.Когда эти две величины нанесены вместе, ток достигает пика на четверть цикла (или π / 2radπ / 2rad) перед напряжением, как показано на рисунке 15.7 (b). Ток через конденсатор опережает напряжение на конденсаторе на π / 2rad, π / 2rad или четверть цикла.

    Соответствующая векторная диаграмма показана на рисунке 15.8. Здесь взаимосвязь между iC (t) iC (t) и vC (t) vC (t) представлена ​​наличием их векторов, вращающихся с одинаковой угловой частотой, с текущим вектором впереди на π / 2rad.π / 2рад.

    Фигура 15,8 Векторная диаграмма конденсатора на рисунке 15.7. Вектор тока опережает вектор напряжения на π / 2π / 2 рад, поскольку они оба вращаются с одинаковой угловой частотой.

    До сих пор в нашем обсуждении мы использовали исключительно пиковые значения тока или напряжения, а именно I0I0 и V0.V0. Однако, если мы усредним значения тока или напряжения, эти значения равны нулю. Поэтому мы часто используем второе соглашение, называемое среднеквадратичным значением или среднеквадратичным значением, при обсуждении тока и напряжения.RMS работает в обратном порядке по отношению к терминологии. Сначала вы возводите функцию в квадрат, затем вы берете среднее значение, а затем находите квадратный корень. В результате среднеквадратичные значения тока и напряжения не равны нулю. Для приборов и устройств обычно указываются среднеквадратические значения для их работы, а не пиковые значения. Мы указываем среднеквадратичные значения с помощью нижнего индекса, прикрепленного к заглавной букве (например, IrmsIrms).

    Хотя конденсатор в основном представляет собой разомкнутую цепь, среднеквадратичный ток или среднеквадратическое значение тока появляется в цепи с переменным напряжением, приложенным к конденсатору.Считаем, что

    , где I0I0 — пиковый ток в системе переменного тока. Среднеквадратичное значение напряжения или среднеквадратичное значение напряжения равно

    .

    , где V0V0 — пиковое напряжение в системе переменного тока. Среднеквадратичный ток появляется потому, что напряжение постоянно меняет направление, заряжает и разряжает конденсатор. Если частота стремится к нулю, что может быть постоянным напряжением, XCXC стремится к бесконечности, и ток равен нулю после зарядки конденсатора. На очень высоких частотах реактивное сопротивление конденсатора стремится к нулю — он имеет незначительное реактивное сопротивление и не препятствует току (он действует как простой провод).

    Индуктор

    Наконец, давайте рассмотрим катушку индуктивности, подключенную к источнику переменного напряжения. Согласно правилу петли Кирхгофа, напряжение на катушке индуктивности L на рис. 15.9 (а) равно

    . vL (t) = V0sinωt.vL (t) = V0sinωt.

    15,6

    ЭДС на катушке индуктивности равна ε = −L (diL / dt); ε = −L (diL / dt); однако разность потенциалов на катушке индуктивности равна vL (t) = LdiL (t) / dtvL (t) = LdiL (t) / dt, потому что, если мы примем во внимание, что напряжение вокруг контура должно быть равно нулю, напряжение, полученное от Источник переменного тока должен рассеиваться через катушку индуктивности.Следовательно, соединив это с источником переменного напряжения, мы имеем

    diL (t) dt = V0Lsinωt.diL (t) dt = V0Lsinωt. Фигура 15,9 (а) Катушка индуктивности, подключенная к генератору переменного тока. (b) Ток iL (t) iL (t) через катушку индуктивности и напряжение vL (t) vL (t) на катушке индуктивности. Здесь iL (t) iL (t) отстает от vL (t) vL (t) на π / 2π / 2 рад.

    Текущее значение iL (t) iL (t) находится путем интегрирования этого уравнения. Поскольку в цепи нет источника постоянной ЭДС, в цепи отсутствует установившийся ток.Следовательно, мы можем установить постоянную интегрирования, которая представляет собой установившийся ток в цепи, равным нулю, и мы имеем

    iL (t) = — V0ωLcosωt = V0ωLsin (ωt − π2) = I0sin (ωt − π2), iL (t) = — V0ωLcosωt = V0ωLsin (ωt − π2) = I0sin (ωt − π2),

    15,7

    , где I0 = V0 / ωL.I0 = V0 / ωL. Связь между V0V0 и I0I0 также может быть записана в форме, аналогичной закону Ома:

    V0I0 = ωL = XL.V0I0 = ωL = XL.

    15,8

    Величина XLXL известна как индуктивное реактивное сопротивление катушки индуктивности или сопротивление катушки индуктивности изменению тока; его единица — тоже ом.Обратите внимание, что XLXL напрямую зависит от частоты источника переменного тока — высокая частота вызывает высокое индуктивное сопротивление.

    Между текущим током и напряжением на катушке индуктивности возникает разность фаз π / 2π / 2 рад. Согласно уравнениям 15.6 и 15.7, ток через катушку индуктивности отстает от разности потенциалов на катушке индуктивности на π / 2radπ / 2rad, или на четверть цикла. Векторная диаграмма для этого случая показана на рисунке 15.10.

    Фигура 15.10 Векторная диаграмма катушки индуктивности на рисунке 15.9. Вектор тока отстает от вектора напряжения на π / 2π / 2 рад, поскольку они оба вращаются с одинаковой угловой частотой.

    Пример 15.1

    Простые цепи переменного тока
    Генератор переменного тока создает ЭДС амплитудой 10 В на частоте f = 60 Гц. F = 60 Гц. Определите напряжения и токи через элементы схемы, когда генератор подключен к (a) резистору 100 Ом и 100 Ом, (b) конденсатору 10 мкФ10 мкФ и (c) катушке индуктивности 15 мГн.
    Стратегия
    Полное переменное напряжение на каждом устройстве совпадает с напряжением источника.Мы можем найти токи, найдя реактивное сопротивление X каждого устройства и вычислив пиковый ток, используя I0 = V0 / X.I0 = V0 / X.
    Решение
    Напряжение на выводах источника равно v (t) = V0sinωt = (10V) sin120πt, v (t) = V0sinωt = (10V) sin120πt,

    , где ω = 2πf = 120πrad / sω = 2πf = 120πrad / s — угловая частота. Поскольку v ( t ) также является напряжением на каждом из элементов, мы имеем

    v (t) = vR (t) = vC (t) = vL (t) = (10V) sin120πt.v (t) = vR (t) = vC (t) = vL (t) = (10V) sin120πt.

    а. Когда R = 100 Ом, R = 100 Ом, амплитуда тока через резистор составляет

    . I0 = V0 / R = 10 В / 100 Ом = 0,10 А, I0 = V0 / R = 10 В / 100 Ом = 0,10 А,

    т.

    iR (t) = (0,10A) sin120πt.iR (t) = (0,10A) sin120πt.

    г. Из уравнения 15.3 емкостное реактивное сопротивление равно

    . XC = 1ωC = 1 (120πрад / с) (10 × 10−6F) = 265Ω, XC = 1ωC = 1 (120πrad / s) (10 × 10−6F) = 265Ω,

    , поэтому максимальное значение тока равно

    I0 = V0XC = 10V265Ω = 3,8 × 10−2AI0 = V0XC = 10V265Ω = 3,8 × 10−2A

    , а мгновенный ток —

    . iC (t) = (3.8 × 10−2A) sin (120πt + π2). IC (t) = (3.8 × 10−2A) sin (120πt + π2).

    г. Из уравнения 15.8 индуктивное реактивное сопротивление равно

    . XL = ωL = (120πрад / с) (15 × 10–3H) = 5,7 Ом. XL = ωL = (120πрад / с) (15 × 10–3H) = 5,7 Ом.

    Таким образом, максимальный ток равен

    . I0 = 10 В 5,7 Ом = 1,8 A I0 = 10 В 5,7 Ом = 1,8 А

    и мгновенный ток

    iL (t) = (1.8A) sin (120πt − π2). iL (t) = (1.8A) sin (120πt − π2).
    Значение
    Хотя напряжение на каждом устройстве одинаково, пиковый ток имеет разные значения в зависимости от реактивного сопротивления.Реактивное сопротивление каждого устройства зависит от значений сопротивления, емкости или индуктивности.

    Проверьте свое понимание 15,2

    Проверьте свое понимание Повторите пример 15.1 для источника переменного тока с амплитудой 20 В и частотой 100 Гц.

    ПРИНЦИПЫ ПЕРЕМЕННОГО ТОКА

    Щелкните или коснитесь приведенных ниже примеров схем, чтобы вызвать TINACloud, и выберите интерактивный режим постоянного тока, чтобы проанализировать их в режиме онлайн.
    Получите недорогой доступ к TINACloud для редактирования примеров или создания собственных схем ПАССИВНЫЕ КОМПОНЕНТЫ В ЦЕПИ ПЕРЕМЕННОГО ТОКА

    Синусоидальное напряжение можно описать уравнением:

    v (t) = V M sin (ωt + Φ) или v (t) = V M cos (ωt + Φ)

    где v (t) Мгновенное значение напряжения в вольтах (V).
    В M Максимальное или пиковое значение напряжения в вольтах (В)
    T Период: время, затраченное на один цикл, в секундах
    f Частота — количество периодов в 1 секунде, в Гц (Герцах) или 1 / с. f = 1 / T
    ω Угловая частота, выраженная в радианах / с
    ω = 2 * π * f или ω = 2 * π / T.
    Φ Начальная фаза даны в радианах или градусах.Эта величина определяет значение синусоидальной или косинусоидальной волны att = 0.
    Примечание. Амплитуда синусоидального напряжения иногда выражается как V Eff , эффективное или среднеквадратичное значение. Это связано с V M в соответствии с соотношением V M = √2V Eff, или приблизительно V Eff = 0,707 V M

    Вот несколько примеров, иллюстрирующих приведенные выше термины. .

    Характеристики напряжения 220 В переменного тока в бытовых электрических розетках в Европе:

    Эффективное значение: В Eff = 220 В
    Пиковое значение: В M = √2 * 220 В = 311 В

    Частота: f = 50 1 / с = 50 Гц
    Угловая частота: ω = 2 * π * f = 314 1 / с = 314 рад / с
    Период: T = 1 / f = 20 мс
    Функция времени: v (t) = 311 sin (314 t)

    Давайте посмотрим на функцию времени, используя команду TINA Analysis / AC Analysis / Time Function.

    Щелкните / коснитесь схемы выше, чтобы проанализировать в режиме онлайн, или щелкните эту ссылку, чтобы сохранить в Windows

    . Вы можете проверить, что период равен T = 20 м, а V M = 311 В.

    Характеристики напряжения 120 В переменного тока в бытовой электросети в США:

    Эффективное значение: В Eff = 120 В
    Пиковое значение: В M = √2 120 В = 169,68 В ≈ 170 В
    Частота: f = 60 1 / с = 60 Гц
    Угловая частота: ω = 2 * π * f = 376,8 рад / с ≈ 377 рад / с
    Период: T = 1 / f = 16,7 мс
    Функция времени: v (t) = 170 sin (377 t)

    Обратите внимание, что в этом случае функция времени может быть задана как v (t) = 311 sin (314 t + Φ) или v (t) = 311 cos (314 t + Φ), так как в случае выходного напряжения мы не знаем начальную фазу.

    Начальная фаза играет важную роль, когда одновременно присутствует несколько напряжений. Хорошим практическим примером является трехфазная система, в которой присутствуют три напряжения с одинаковым пиковым значением, формой и частотой, каждое из которых имеет фазовый сдвиг на 120 ° относительно других. В сети 60 Гц функции времени:

    v A (t) = 170 sin (377 t)

    v B (t) = 170 sin (377 t — 120 °)

    v C (t) = 170 sin (377 t + 120 °)

    На следующем рисунке, сделанном с помощью TINA, показана схема с этими временными функциями в качестве генераторов напряжения TINA.


    Щелкните / коснитесь схемы выше, чтобы проанализировать в режиме онлайн, или щелкните эту ссылку, чтобы сохранить в Windows

    Разница напряжений v AB = v A (t) — v B (t) показана как решенная с помощью Команда TINA Analysis / AC Analysis / Time Function.

    Обратите внимание, что пик напряжения v AB (t) составляет примерно 294 В, что больше, чем пиковое значение 170 В напряжений v A (t) или v B (t), но также не является просто суммой их пиковое напряжение. Это связано с разностью фаз.Мы обсудим, как рассчитать результирующее напряжение (которое в данном случае составляет Ö3 * 170 @ 294) позже в этой главе, а также в отдельной главе Трехфазные системы .

    Характеристические значения синусоидальных сигналов

    Хотя сигнал переменного тока непрерывно изменяется в течение своего периода, легко определить несколько характерных значений для сравнения одной волны с другой: это пиковое, среднее и среднеквадратичное ( среднеквадратичные) значения.

    Мы уже встретили пиковое значение V M , которое является просто максимальным значением временной функции, амплитудой синусоидальной волны.

    Иногда используется значение размаха (размах). Для синусоидальных напряжений и токов величина размаха вдвое превышает пиковое значение.

    Среднее значение синусоиды — это среднее арифметическое значений для положительного полупериода. Его также называют абсолютным средним значением , поскольку оно совпадает со средним абсолютным значением сигнала. На практике мы сталкиваемся с этой формой волны, когда выпрямляет синусоидальную волну с помощью схемы, называемой двухполупериодным выпрямителем.

    Можно показать, что абсолютное среднее значение синусоидальной волны составляет:

    В AV = 2 / π V M ≅ 0,637 В M

    Обратите внимание, что среднее значение всего цикла равно нулю.
    Действующее или действующее значение синусоидального напряжения или тока соответствует эквивалентному значению постоянного тока, обеспечивающему такую ​​же мощность нагрева. Например, напряжение с эффективным значением 120 В создает такую ​​же мощность нагрева и освещения в лампочке, что и 120 В от источника постоянного напряжения.Можно показать, что действующее или действующее значение синусоидальной волны составляет:

    В действующее значение = В M / √2 ≅ 0,707 В M

    Эти значения могут быть рассчитаны одинаково для обоих напряжений и токи.

    Среднеквадратичное значение очень важно на практике. Если не указано иное, напряжение переменного тока в линии питания (например, 110 В или 220 В) дано в среднеквадратических значениях. Большинство измерителей переменного тока откалиброваны в среднеквадратичном значении и показывают среднеквадратичный уровень.

    Пример 1 Найдите пиковое значение синусоидального напряжения в электрической сети со среднеквадратичным значением 220 В.

    В M = 220 / 0,707 = 311,17 В

    Пример 2 Найдите пиковое значение синусоидального напряжения в электрической сети со среднеквадратичным значением 110 В.

    В M = 110 / 0,707 = 155,58 В

    Пример 3 Найдите (абсолютное) среднее синусоидальное напряжение, если его действующее значение составляет 220 В.

    В a = 0,637 * В M = 0,637 * 311,17 = 198,26 В

    Пример 4 Найдите абсолютное среднее значение синусоидального напряжения, если его действующее значение составляет 110 В.

    Пик напряжения из Примера 2 составляет 155,58 В и, следовательно:

    В a = 0,637 * В M = 0,637 * 155,58 = 99,13 В

    Пример 5 Найдите соотношение между абсолютным средним (V a ) и среднеквадратичным (V) значениями для синусоидального сигнала.

    В / В a = 0,707 / 0,637 = 1,11

    Обратите внимание, что нельзя складывать средние значения в цепи переменного тока, поскольку это приводит к неверным результатам.

    ФАЗОРЫ

    Как мы уже видели в предыдущем разделе, в цепях переменного тока часто необходимо добавлять синусоидальные напряжения и токи той же частоты. Хотя можно добавить сигналы численно с помощью TINA или с помощью тригонометрических соотношений, более удобно использовать так называемый метод векторов . Вектор — это комплексное число, представляющее амплитуду и фазу синусоидального сигнала. Важно отметить, что вектор не представляет частоту, которая должна быть одинаковой для всех векторов.

    С вектором можно обращаться как с комплексным числом или графически представлять в виде плоской стрелки на комплексной плоскости. Графическое изображение называется векторной диаграммой. Используя векторные диаграммы, вы можете складывать или вычитать векторы в комплексной плоскости по правилу треугольника или параллелограмма.

    Есть две формы комплексных чисел: прямоугольное и полярное .

    Прямоугольное представление имеет вид a + j b, где j = Ö-1 — мнимая единица.

    Полярное представление имеет вид Ae j j , где A — абсолютное значение (амплитуда), а f — угол вектора относительно положительной вещественной оси в направлении против часовой стрелки.

    Мы будем использовать полужирным шрифтом, букв для сложных количеств.

    Теперь давайте посмотрим, как получить соответствующий вектор из функции времени.

    Во-первых, предположим, что все напряжения в цепи выражены в виде функций косинуса.(Все напряжения могут быть преобразованы в эту форму.) Тогда вектор , соответствующий напряжению v (t) = V M cos (w t + f), равен: V M = V M e jf , которое также называют комплексным пиковым значением.

    Например, рассмотрим напряжение: v (t) = 10 cos (w t + 30 °)

    Соответствующий вектор: V

    Таким же образом мы можем вычислить функцию времени из вектора. Сначала запишем вектор в полярной форме e.грамм. V M = V M e j r и тогда соответствующая функция времени будет

    v (t) = V M (cos (wt + r).

    Например , рассмотрим вектор V M = 10 — j 20 V

    Приведем его к полярной форме:

    И, следовательно, функция времени: v (t) = 22,36 cos (wt — 63,5 ° ) V

    Фазоры часто используются для определения комплексного действующего или действующего значения напряжений и токов в цепях переменного тока.Дано v (t) = V M cos (wt + r) = 10cos (wt + 30 °)

    Численно:

    v (t) = 10 * cos (wt-30 °)

    Комплекс эффективное (среднеквадратичное) значение: В = 0,707 * 10 * e j 30 ° = 7,07 e j 30 ° = 6,13 — j 3,535

    Наоборот: если комплексное эффективное значение напряжения:

    В = — 10 + j 20 = 22.36 e j 116,5 °

    , затем комплексное пиковое значение:

    и функция времени: v (t) = 31,63 cos (wt + 116,5 °) V

    Краткое обоснование вышеуказанных методов как следует. Учитывая временную функцию
    V M (cos (w t + r), давайте определим комплексную временную функцию как:

    v (t) = V M e j r e j wt = V M e j wt = V M (cos (r) + j sin (r)) e j wt

    где V M = V M e j r t = V M (cos (r) + j sin (r)) просто вектор, введенный выше.

    Например, комплексная функция времени v (t) = 10 cos (wt + 30 °)

    v (t) = V M e j wt = 10 e j 30 e j wt = 10e j wt (cos (30) + j sin (30)) = e j wt wt 8.66+ j 5)

    Введя комплексную функцию времени, мы получаем представление как с действительной, так и с мнимой частями.Мы всегда можем восстановить исходную реальную функцию времени, взяв действительную часть нашего результата: v (t) = Re { v (t)}

    Однако комплексная функция времени имеет большое преимущество, поскольку, поскольку все сложные Временные функции в рассматриваемых цепях переменного тока имеют один и тот же множитель e j w t , мы можем это вычесть и просто работать с векторами. Более того, на практике мы вообще не используем часть e j w t — только преобразования из функций времени в векторы и обратно.

    Чтобы продемонстрировать преимущества использования векторов, рассмотрим следующий пример.

    Пример 6 Найдите сумму и разность напряжений:

    v 1 = 100 cos (314 * t) и v 2 = 50 cos (314 * t-45 °)

    Сначала напишите векторы обоих напряжений:

    V 1M = 100 V 2M = 50 e j 45 ° = 35,53 — j 35.35

    Отсюда:

    V добавить = V 1M + V 2M = 135,35 — j 35,35 = 139,89 e — j 14,63 6 9 °

    V sub = V 1M V 2M = 64,65 + j 35,35 = 73,68 e j 28,67 °

    и затем функции времени: v

    87 90 прибавляем (t) = 139.89 * cos (wt — 14,63 °)

    v sub (t) = 73,68 * cos (wt + 28,67 °)

    Как показывает этот простой пример, метод векторов является чрезвычайно мощным инструментом для решения проблем с переменным током .

    Давайте решим проблему с помощью инструментов в интерпретаторе TINA.

    {расчет v1 + v2}
    v1: = 100
    v2: = 50 * exp (-pi / 4 * j)
    v2 = [35.3553-35.3553 * j]
    v1add: = v1 + v2
    v1add = [135,3553-35,3553 * j]
    абс (v1add) = [139,8966]
    радтодег (arc (v1add)) = [- 14.6388]

    {расчет v1-v2}
    v1sub: = v1-v2
    v1sub = [64.6447 + 35.3553 * j]
    abs (v1sub) = [73.6813]
    radtodeg (arc (v1sub)) = [28.6751]

    907

    Результаты амплитуды и фазы подтверждают ручные вычисления.

    Теперь давайте проверим результат с помощью анализа переменного тока TINA.

    Перед выполнением анализа убедитесь, что для базовой функции для AC ia задано значение , косинус в диалоговом окне «Параметры редактора » в меню «Вид / Параметры».Мы объясним роль этого параметра в , Пример 8 .

    Схемы и результаты:

    Щелкните / коснитесь схемы выше, чтобы проанализировать в режиме онлайн, или щелкните эту ссылку, чтобы сохранить под Windows

    И снова результат тот же. Вот графики функции времени:


    Пример 7 Найдите сумму и разность напряжений:

    v 1 = 100 sin (314 * t) и v 2 = 50 cos (314 * t- 45 °)

    В этом примере возникает новый вопрос.До сих пор мы требовали, чтобы все функции времени были заданы как функции косинуса. Что нам делать с функцией времени, заданной как синус? Решение состоит в том, чтобы преобразовать функцию синуса в функцию косинуса. Используя тригонометрическое соотношение sin (x) = cos (xp / 2) = cos (x-90 °), наш пример можно перефразировать следующим образом:

    v 1 = 100 cos (314t — 90 °) и v 2 = 50 cos (314 * t — 45 °)

    Теперь векторы напряжений:

    V 1M = 100 e j 90 ° = -100 j V 2M = 50 e j 45 ° = 35.53 — j 35,35

    Отсюда:

    V добавить = V 1M + V 2M = 35,53 — j 135,35

    9007 = V 1M V 2M = — 35,53 — j 64,47

    , а затем функции времени:

    v добавить (t) = 139,8966 cos (wt-75,36 °)

    v sub (t) = 73.68 cos (wt-118,68 °)

    Давайте решим проблему с помощью инструментов в интерпретаторе TINA.

    {расчет v1 + v2}
    v1: = — 100 * j
    v2: = 50 * exp (-pi / 4 * j)
    v2 = [35,3553 — 35,3553 * j]
    v1add: = v1 + v2
    v1add = [35.3553-135.3553 * j]
    abs (v1add) = [139.8966]
    radtodeg (arc (v1add)) = [- 75.3612]

    {расчет v1-v2}
    v1sub: = v1- v2
    v1sub = [- 35,3553 — 64,6447 * j]
    abs (v1sub) = [73,6813]
    radtodeg (arc (v1sub)) = [- 118.6751]

    Давайте проверим результат с помощью TINA AC Analysis

    Щелкните / коснитесь схемы выше, чтобы проанализировать в режиме онлайн, или щелкните эту ссылку, чтобы сохранить в Windows

    Пример 8
    Найдите сумму и разницу напряжений:

    v 1 = 100 sin (314 * t) и v 2 = 50 sin (314 * t-45 °)

    Этот пример поднимает еще одну проблему. Что, если все напряжения даны как синусоидальные волны, и мы также хотим видеть результат как синусоидальную волну?Конечно, мы могли бы преобразовать оба напряжения в функции косинуса, вычислить ответ, а затем преобразовать результат обратно в функцию синуса, но в этом нет необходимости. Мы можем создать фазоры из синусоидальных волн так же, как и из косинусоидальных волн, а затем просто использовать их амплитуду и фазы как амплитуду и фазу синусоидальных волн в результате.

    Очевидно, это даст тот же результат, что и преобразование синусоидальных волн в косинусоидальные. Как мы могли видеть в предыдущем примере, это эквивалентно умножению на — j , а затем использованию отношения cos (x) = sin (x-90 °) для преобразования его обратно в синусоидальную волну.Это эквивалентно умножению на j . Другими словами, поскольку — j × j = 1, мы могли бы использовать векторы, полученные непосредственно из амплитуд и фаз синусоидальных волн, для представления функции, а затем вернуться к ним напрямую. Кроме того, рассуждая таким же образом о сложных функциях времени, мы могли бы рассматривать синусоидальные волны как мнимые части комплексных функций времени и дополнять их функцией косинуса для создания полной комплексной функции времени.

    Давайте посмотрим на решение этого примера, используя синусоидальные функции в качестве основы векторов (преобразование sin (w t) в реальный единичный вектор (1)).

    V 1M = 100 V 2M = 50 e j 45 ° = 35,53 — j 35,35

    Отсюда:

    90

    90 = V 1M + V 2M = 135,53 — j 35.35

    V sub = V 1M V 2M = 64,47+ j 35,35

    Обратите внимание, что векторы точно такие же, как в примере 6, но не время функции:

    v 3 (t) = 139.9sin (wt — 14,64 °)

    v 4 (t) = 73.68sin (wt + 28.68 °)

    Как видите, это очень просто получить результат, используя синусоидальные функции, особенно когда наши исходные данные являются синусоидальными волнами.Многие учебники предпочитают использовать синусоидальную волну в качестве базовой функции векторов. На практике вы можете использовать любой метод, но не путайте их.

    При создании векторов очень важно, чтобы все временные функции сначала были преобразованы либо в синус, либо в косинус. Если вы начали с синусоидальных функций, ваши решения должны быть представлены синусоидальными функциями при возврате от векторов к функциям времени. То же самое верно, если вы начнете с функций косинуса.

    Давайте решим ту же задачу, используя интерактивный режим TINA.Поскольку мы хотим использовать синусоидальные функции в качестве основы для создания векторов, убедитесь, что для базовой функции для AC установлено значение синус в диалоговом окне Editor Options из меню View / Option.



    Схемы для суммирования и разности форм сигналов и результата:


    и функции времени:


    ПАССИВНЫЕ КОМПОНЕНТЫ В ЦЕПИ ПЕРЕМЕННОГО ТОКА

    Среднеквадратичное значение тока в цепи переменного тока составляет 10 А Что физика класса 12 CBSE

    Подсказка: Мы уже изучили вывод для соотношения между пиковым значением переменного тока и среднеквадратичным значением переменного тока в главе «Переменный ток устройства 4 в классе 12».Поскольку мы уже указали среднеквадратичное значение тока. Просто замените числовые значения, приведенные в приведенной ниже математической формуле, чтобы найти запрашиваемое значение.

    Полный пошаговый ответ:
    Учитывая, что среднеквадратичное значение тока в цепи переменного тока составляет 10 А. Теперь среднеквадратичное значение, т. Е. Среднеквадратичное значение, которое также известно как эффективное значение переменного тока или виртуальное значение Переменный ток. Мы можем определить его как такое значение постоянного тока (или постоянного тока), которое будет генерировать такое же количество тепла, как и переменным током в той же цепи в течение всего периода времени.

    Простыми словами, мы можем сказать, что количество тепла, произведенного из-за переменного тока в цепи A, тогда такое же количество тепла должно быть произведено в другой цепи B из-за постоянного тока, и это значение постоянного тока известно как среднеквадратичное значение переменного тока. Он представлен как \ [{{\ text {I}} _ \ nu} \] или \ [{{\ text {I}} _ {effective}} \] или \ [{{\ text {I}} _ {RMS}} \]. Путем вывода мы пришли к математическому выражению как:
    \ [{{\ text {I}} _ {RMS}} = \ dfrac {{{I_0}}}} {{\ sqrt 2} } \] ……………………………………… Ур.1
    Где, \ [{{\ text {I}} _ 0} \] = пиковый ток или максимальный ток.
    \ [{{\ text {I}} _ {RMS}} \] = Среднеквадратичное значение переменного тока.
    Подставляя значение \ [\ dfrac {1} {{\ sqrt 2}} = 0.7071 \] в формулу выше, мы получаем
    \ [{{\ text {I}} _ {RMS}} = 0.707 {{\ text {I}} _ 0} \] ……………………………. Eq.2
    Нам нужно найти значение пикового тока. Пиковый ток также называется максимальным током.
    Подставьте числовое значение \ [{\ text {I}} _ {RMS}} \] из вопроса в уравнение 1 или 2,
    \ [
    10 = 0.707 {{\ text {I}} _ 0} \\
    \ Rightarrow \ dfrac {{10}} {{0.707}} = {{\ text {I}} _ 0} \\
    \ Rightarrow 14.144 = {{\ text {I}} _ 0} \\
    \]
    \ [\ поэтому {{\ text {I}} _ 0} = 14,144 \] прибл. \ [{{\ text {I}} _ 0} = 14,1 \] A
    Пиковое значение переменного тока составляет 14,1 ампер в цепи, где действующее значение тока составляет 10 ампер.

    Итак, вариант А верен.

    Примечание: Всегда указывайте единицы физического количества в конце решения. Среднеквадратичное значение любой физической величины никогда не может быть отрицательным.Кроме того, в среднеквадратичном значении мы рассматривали полный цикл переменного тока вместо полупериода, который мы рассматриваем в среднем значении переменного тока. Для большей ясности перейдите от вывода.

    Лаборатория 9 — Схемы переменного тока

    Введение

    Изучение переменного тока (AC) в физике очень важно, поскольку оно имеет практическое применение в нашей повседневной жизни. Как следует из названия, направление тока и напряжения периодически меняются с фиксированной частотой.В США эта частота обычно составляет 60 Гц, а в странах Европы — 50 Гц. Переменный ток передается при высоком напряжении и малом токе по основным линиям передачи, так что потери мощности могут быть сведены к минимуму. Напряжение понижается в несколько раз, прежде чем оно попадет в ваш дом. Большинству электроприборов требуется напряжение 120 В, хотя некоторым приборам, таким как оборудование для обогрева и кондиционирования воздуха, водонагреватели и духовки, требуется 240 В. Ранее в этом курсе вы, возможно, изучали поведение резисторов, конденсаторов и катушек индуктивности в цепи с источником постоянного тока или, как в случае разряда конденсатора, без источника вообще.В этой лабораторной работе вы проанализируете взаимосвязь между током и напряжением, когда эти элементы подключены в цепь с источником ЭДС переменного тока.

    Обсуждение принципов

    Напряжение источника переменного тока изменяется во времени синусоидально с частотой f , задаваемой формулой где

    ω

    — угловая частота, измеряемая в радианах в секунду. Ток, подаваемый источником переменного тока, также является синусоидальным. где I 0 — максимальный или пиковый ток, а t — время.

    Сопротивление в цепи переменного тока

    Рассмотрим резистор R , подключенный к источнику ЭДС переменного тока, как показано на рис. 1 (а).

    Рисунок 1 : Ток и напряжение в резисторе, подключенном к источнику переменного тока с ЭДС

    Ток через резистор определяется формулой. (2)

    I = I 0 cos (2 π ft)

    и напряжение на резисторе подчиняется закону Ома.

    (3)

    V = IR = I 0 cos (2 π футов) R = V 0 cos (2 π футов)

    где пиковое напряжение на резисторе составляет

    В 0 = I 0 Р.

    На рис. 1 (b) показаны графики зависимости тока и напряжения от времени t . Обратите внимание, что и ток, и напряжение достигают пикового значения в один и тот же момент времени, и оба они равны нулю в один и тот же момент времени. Поэтому мы говорим, что ток и напряжение совпадают по фазе или разность фаз равна нулю.

    Индуктивность в цепи переменного тока

    Рассмотрим катушку индуктивности L , подключенную к источнику ЭДС переменного тока, как показано на рис. 2 (а).

    Рисунок 2 : Ток и напряжение в катушке индуктивности, подключенной к источнику переменного тока с ЭДС

    Ток в цепи определяется формулой.(2)

    I = I 0 cos (2 π ft)

    , а напряжение на катушке индуктивности определяется выражением Это ЭДС, индуцированная в катушке из-за изменения тока. Комбинируя уравнение. (2)

    I = I 0 cos (2 π футов)

    и уравнение. (4) дает

    (5)

    В L = −V 0 sin (2 π футов)

    где

    В 0

    — пиковое напряжение. На рис. 2 (b) показаны графики зависимости тока и напряжения от времени t .В этом случае, когда напряжение имеет положительный максимум, ток равен нулю и находится в процессе увеличения. Когда напряжение является отрицательным максимумом, ток уменьшается. Ток достигает своего максимального значения за 1/4 цикла после того, как напряжение достигнет максимума. Поэтому мы говорим, что напряжение опережает ток на 90 ° или ток отстает от напряжения на 90 °. Уравнение (5)

    V L = −V 0 sin (2 π ft)

    можно записать как

    (6)

    В L = В 0 cos (2 π футов + φ )

    где разность фаз

    φ = π /2.

    Ток и напряжение связаны уравнением, аналогичным закону Ома, с где

    X L

    известно как индуктивное реактивное сопротивление, измеряется в омах и определяется выражением

    Емкость в цепи переменного тока

    Рассмотрим конденсатор C , подключенный к источнику ЭДС переменного тока, как показано на рис. 3 (а).

    Рисунок 3 : Ток и напряжение в конденсаторе, подключенном к источнику переменного тока с ЭДС

    Ток в цепи определяется формулой.(2)

    I = I 0 cos (2 π ft)

    , а напряжение на конденсаторе определяется выражением где заряд Q на обкладках конденсатора связан с током соотношением Комбинируя уравнения. (2)

    I = I 0 cos (2 π ft)

    , (9) и (10), получаем

    (11)

    V C = V 0 sin (2 π футов)

    где

    В 0

    — пиковое напряжение. На рис. 3 (b) показаны графики зависимости тока и напряжения от времени t .В этом случае ток достигает максимального значения за 1/4 цикла до того, как напряжение достигнет максимума. Поэтому мы говорим, что напряжение отстает от тока на 90 ° или ток опережает напряжение на 90 °. Напряжение можно записать как

    (12)

    В L = В 0 cos (2 π футов — φ )

    где разность фаз

    φ = π /2.

    Ток и напряжение связаны уравнением, аналогичным закону Ома, с где X C известно как емкостное реактивное сопротивление, определяемое по формуле Здесь X C измеряется в омах.

    Цепь RLC

    Базовая схема RLC показана на рис. 4 с последовательно включенными емкостью C , индуктором L и резистором R .

    Рисунок 4 : Простая схема RLC

    Напряжение от источника переменного тока определяется выражением

    (15)

    V = V R + V C + V L .

    Обратите внимание, что пиковое напряжение

    В 0

    источника не будет равняться сумме индивидуальных пиковых напряжений трех элементов.Это связано с тем, что напряжения на резисторе, конденсаторе и катушке индуктивности не достигают своих пиковых значений одновременно. Полный импеданс Z цепи RLC определяется выражением

    (16)

    Z =
    (X C — X L ) 2 + R 2
    .

    Закон Ома для этой цепи, а также для каждого компонента следующий. Приложенное напряжение и ток обычно не совпадают по фазе на , а на разность фаз равна

    (21)

    φ = загар −1
    . Поскольку импеданс зависит от частоты, ток также зависит от частоты, даже если амплитуда напряжения постоянна. Согласно формуле. (17)

    В 0 = I 0 Z

    , ток будет максимальным, когда полное сопротивление будет минимальным. Это происходит, когда

    X L — X C = 0,

    , т.е. когда

    (22)

    ω =
    1
    LC
    . Эта частота, при которой ток максимален, известна как резонансная частота цепи. Рис. 5 представляет собой график зависимости тока от частоты для конкретного значения R .

    Рисунок 5 : График зависимости тока от частоты

    На резонансной частоте сопротивление чисто резистивное. Для очень малых значений R цепь называется LC-цепью, и энергия колеблется между катушкой индуктивности и конденсатором.Конечно, через резистор будет рассеиваться немного энергии. Применение явления резонансной частоты включает радио и телевизоры, в которых для настройки на определенную станцию ​​используется различное значение L или C .

    Цель

    Целью этого эксперимента является изучение резонанса в последовательной цепи RLC путем измерения и построения графика амплитуды напряжения в зависимости от частоты, а также для наблюдения изменения фазового угла для значений частоты по обе стороны от резонансного значения.

    Оборудование

    • Печатная плата PASCO
    • Программное обеспечение Capstone
    • Сигнальный интерфейс с выходной мощностью
    • Соединительные провода
    • Мультиметр

    Процедура

    Распечатайте лист для этой лабораторной работы. Этот лист понадобится вам для записи ваших данных.

    Процедура A: Определение резонансной частоты

    1

    Выключите усилитель мощности и интерфейс сигнала.

    2

    Постройте схему, показанную на рис.6.

    Рисунок 6 : Принципиальная электрическая схема

    3

    Вставьте железный сердечник в центр индуктора. Это увеличивает индуктивность катушки. Для конкретной печатной платы, которую вы будете использовать в этой лаборатории, индуктивность катушки (со вставленным железным сердечником) составляет 33 мГн.

    4

    В качестве источника напряжения будет использоваться усилитель мощности. Подключите его выходные клеммы к резистору и конденсатору, как показано, с помощью соединительных шнуров с банановой вилкой.

    5

    Вставьте DIN-штекер датчика напряжения в аналоговый канал A сигнального интерфейса и подключите его выводы через резистор 10 Ом. Напряжение, измеренное на аналоговом канале A, будет использоваться для расчета тока I , который связан с напряжением

    В R

    на резисторе.См. Уравнение. (3)

    V = IR = I 0 cos (2 π футов) R = V 0 cos (2 π футов)

    .

    Рисунок 7 : Подробная информация о соединениях

    Контрольная точка 1:
    Попросите своего технического специалиста проверить вашу схему.

    6

    Включите компьютер и монитор, сигнальный интерфейс и усилитель мощности.

    7

    Откройте файл Capstone, связанный с этой лабораторной работой, которая запускает программу Capstone.Отобразится экран, аналогичный показанному на рис. 8.

    Рисунок 8 : Начальный экран в Capstone

    8

    Убедитесь, что генератор сигналов настроен на создание синусоидальной волны с частотой 10 Гц и амплитудой 5 вольт.

    Рисунок 9 : Настройки генератора сигналов

    9

    Запишите это как напряжение

    В 0

    на листе.

    10

    Щелкните ON , затем щелкните START , чтобы начать сбор данных.

    11

    Чтобы увидеть резонанс, прокрутите частоты, изменив настройки в окне генератора сигналов. Вы можете сделать это, щелкнув стрелки вверх и вниз или щелкнув число и введя новое значение. По мере того, как вы переходите по частоте, обратите внимание, что

    В R

    увеличивается по мере приближения частоты к резонансной частоте и затем уменьшается по мере увеличения частоты за пределы резонансной частоты. Также обратите внимание, что разность фаз между выходным напряжением усилителя и

    В R

    стремится к нулю на резонансной частоте.

    12

    Используйте инструмент Показать координаты данных, чтобы измерить напряжения на резисторе

    В R

    и источнике

    В 0 .

    Нажмите кнопку и перейдите к точке, которую вы хотите измерить. См. Рис. 10 ниже.

    Рисунок 10 : Снимок экрана, показывающий кнопку Показать координаты данных

    13

    Увеличьте частоту на 10 Гц. Запишите эту частоту на листе и повторите шаг 12.

    14

    Повторяйте шаги 12 и 13, пока не будет достигнута частота 160 Гц. По мере увеличения частоты может потребоваться регулировка скорости развертки и вертикальной шкалы, чтобы получить четкое изображение

    V R

    на экране.

    15

    Посмотрите на полученные данные и приблизительно определите, на какой частоте возник резонанс, посмотрев, где

    V R

    было максимальным. Установите генератор сигналов на эту частоту и выполните точную настройку частоты, пока след тока не будет совпадать по фазе с напряжением.Схема теперь работает на резонансной частоте. Запишите это значение на листе.

    16

    Используя резонансную частоту, рассчитайте резонансную угловую частоту, используя

    ω res = 2 π f res .

    17

    Рассчитайте теоретическую резонансную частоту, используя формулу. (22) ω =
    1
    LC
    .и значения L , C и R .

    18

    Сравните измеренную резонансную частоту с этим теоретическим значением, вычислив погрешность в процентах. См. Приложение Б. Не бросайте Capstone.

    Контрольная точка 2:
    Попросите своего технического специалиста проверить ваши данные и расчеты.

    Процедура B: Определение фазового сдвига

    19

    Щелкните левой кнопкой мыши метку «Напряжение» в левой части окна результатов, затем выберите «Добавить подобное измерение», затем выберите «Выходное напряжение».

    20

    Установите частоту 10 Гц и изучите формы волны. На рис. 11 показаны снимки обоих напряжений.

    Рисунок 11 : Снимок экрана, показывающий выходное напряжение и напряжение резистора

    Определите, возникает ли пик напряжения на резисторе до или после пика выходного напряжения. Возможно, вам будет полезно взять очень небольшой период времени и отрегулировать шкалу осциллографа, чтобы посмотреть на начальные напряжения.

    21

    Если временной интервал между пиком

    В 0

    и пиком

    В R

    составляет

    Δt,

    , то величина разности фаз составляет

    φ = 2 π (fΔt).

    22

    Повторите это, чтобы найти фазовый сдвиг для частоты 160 Гц.

    Контрольная точка 3:
    Попросите своего технического специалиста проверить ваши данные и расчеты.

    Авторские права © 2012 Advanced Instructional Systems, Inc.и государственный университет Северной Каролины | Кредиты

    .

    alexxlab

    Добавить комментарий

    Ваш адрес email не будет опубликован. Обязательные поля помечены *